Этого треда уже нет.
Это копия, сохраненная 10 января 2019 года.

Скачать тред: только с превью, с превью и прикрепленными файлами.
Второй вариант может долго скачиваться. Файлы будут только в живых или недавно утонувших тредах. Подробнее

Если вам полезен архив М.Двача, пожертвуйте на оплату сервера.
Тупых вопросов. Прошлый: https://2ch.hk/sci/res/446142.html 448550 В конец треда | Веб
Тупых вопросов.
Прошлый: https://2ch.hk/sci/res/446142.html (М)
2 448552
>>448512
Например нихуя непонятно с жутким взаимодействием запутанных частиц. При этом экспериментально доказано, что никаких параметров спина (поляризации) у частиц нет, они проявляются только при измерении, тем не менее, результаты этих измерений всегда будут противоположны. Действительно жутко, если начать вдумываться.
3 448555
Почему у человека шерстяной покров головы и мудей седеет, а у всех прочих млекопитающих - нет?
4 448556
>>48555
Седеют. Просто обчно не доживают, до этого момента.
sedye-sobaki-e1493752859628.jpg44 Кб, 700x394
5 448560
Снимок.PNG33 Кб, 1003x581
6 448574
Ребята, почему черные дыры массой 3-1- солнечных масс являются черными дырами и делают свои чернодыровые дела, а огромные звезды пикрелейтед работают как звезды? Они же тяжелее, хули они в чд не превращаются и не засасывают все?
7 448575
>>48574
А кто их в размер меньший горизонта событий для этой массы сожмёт, твой анус?
8 448578
>>48574
давление в ядре препятствует гравитационному сжатию.
Стикер191 Кб, 500x500
9 448580
>>48578
>>48575
Почему говорят мол черная дыра- самый массивный обьект по вселенной? Пиздят оказывается?
10 448581
>>48580
Они есть по 10-20-40 и т.д. миллиардов солнечных масс.
Ты вывод этот на основе таблички со звёздами сделал?
11 448585
>>48574
Все как по Теореме гаусса. Гравитационный потенциал внутри однородного шара равен потенциалу на границе. Так что основной критерий превращения тела в ЧД это его конечные размеры. Если радиус тела больше радиуса шварцшильда, то у тебя не чд.
Тут надо делать оговорку. При учете неоднородностей тела и его момента импульса, то чтобы превратить его в ЧД потребуется его сжать еще меньший размер, чем радиус шварцшильда.
Другое вопрос что удерживает звезду и прочие компактные объекты от сжатия.
Тут либо температура газа, давление вызванное термоядерной реакцией, либо давление вырожденного газа.
12 448593
>>48550 (OP)
хуйня с видео реально работает? можно генератор цеплять?
13 448595
>>48593
ты сможешь объяснить почему она работает?
14 448598
>>48595

ну мож ето гравитационный двихотель
15 448599
>>48598
пока ты не можешь объяснить причину задаваться целью что-то цеплять смысла не имеет. все зацепы на генератор идут через редуктор. не зная параметров перехода с вала ты ничего никуда не зацепишь.
16 448600
Объясните мне, как тупые насекомые научились перенимать окрас и даже имитировать один в один предметы, на которых сидят и при этом понимают это, выработав соответствующее поведение и использующие масировку осознанно. Невозможно сделать это с помощью рандома. Как жук может понимать, что его окраска выглядит угрожающе и отпугивающе или наоборот chrhsdftn его? И как она могла появитсья просто так? Даже растения вырабатывают яд и колючки для защиты, хотя не способны мыслить.
17 448601
>>48600

>как тупые насекомые научились


300 000 000 лет это тебе не хуй собачий
18 448602
>>48601
Это ничего не объясняет. Как они могут понимать свой окрас, если не видят его? Как маскировка стала идентичной веткам и листьям и почему они их имитируют? Как происходит смена цвета по сезонам? Почему они не вымерли ещё до того, как научились этому? Как организмы научились вырабатывать яды для убийства или паралича? Это же невозможно на одном рандоме.
19 448603
>>48602

>Это ничего не объясняет.


тебе и собака ничего не объясняет.

>Это же невозможно на одном рандоме.


каком рандоме блядь когда боженька всех создал.
20 448604
>>48603
Блядь, как же у учёнодаунов всё просто. Вот не было нихуя, потом взрыв из нихуя и всё так само появилось, организовалось, откуда-то законы взялись и жизнь сама зародилась и расплодилась в миллиардах форм и ещё самоосозналась. Просто. Откуда организмы научились смертоносный яд вырабатывать, различать цвета, вырабатывать и плести паутину узором - не важно. Просто всё как-то само по себе завертелось и существует.
21 448607
>>48602
Читай про генетические алгоритмы, про то, как обучаются нейросети. Про эффект бутылочного горлышка. Суть в том, что там далеко не чистый рандом.
22 448608
>>48600
Прости, что поленился объяснять: https://ru.wikipedia.org/wiki/Эволюционные_алгоритмы
23 448610
>>48607
>>48608
Какое, например, условие способствует выработке организмом яда? Даже если это так, то отбор получается осмысленным, а значит имеет задачу и конечную цель.
24 448613
Откуда берётся гравитация? Это ведь бесконечный источник энергии из ничего. Разве это возможно?
25 448615
>>48614 (Del)
Правила не могут быть сами по себе, они задаются кем-то или чем-то с определённой целью.
26 448618
>>48610

> Какое, например, условие способствует выработке организмом яда?


Например, условие наличия хищников.
27 448619
>>48617 (Del)
В смысле аргументы? Очевидно же, что появление сознания не случайность, а результат определённой задачи. Способность развиваться это подтверждает.
28 448621
>>48618
Но яд применяется не против хищников, а в основном для убийства жертв. Но почему-то всё равно есть не у каждого организма. Как можно себе представить внезапное зарождение в организме желёз, которые будут вырабатывать токсин с определённой целью? Откуда эта информация в генах?
29 448622
>>48615
А этот "кто-то" действует по правилам?
30 448624
>>48621

> Но яд применяется не против хищников, а в основном для убийства жертв.


Не знаю, ну считал ни тех, ни других. Ну могу сравнивать кого больше.

> Но почему-то всё равно есть не у каждого организма.


Яз не обязателен для убийства.

> Как можно себе представить внезапное зарождение в организме желёз, которые будут вырабатывать токсин с определённой целью?


Например, в процессе эволюции слюны.

> Откуда эта информация в генах?


Выэволюционирлвала.
31 448625
>>48604

>Блядь, как же у учёнодаунов всё просто.


Это такая самокритика или просто накипело от малолетних ЛеоТаксилистов?

> Вот не было нихуя


WUT? Прямо таки нихуя? Даже в вакууме запасена энергия, если что. А из математического нихуя даже самые отпетые Пенроузы физику выводить не осмеливаются.

> потом взрыв из нихуя


Ну а хули ты хотел, надо же как-то для детворы популизировать. Однако, нихуя, как уже указано выше, нихуя не нихуя, взрыв не то, чтобы совсем взрыв (взрыв пространства-то? для инфляционного периода? звучит забавно, ну ок, имплозия как взрыв имеет право на существование), да и сам процесс не то, чтобы сам процесс, ибо на начальных этапах времени как времени, в общем-то, вероятно и не было.

> и всё так само появилось


Проблемы? Энергия (хоть даже вакуума) не может перейти из одного состояния в другое?

> организовалось


Проблемы? Физические системы лично тебе обязаны быть изолированными друг от друга?

> откуда-то законы взялись


Если критика к конкретным энергетическим параметрам, то загляни в теории Мультиверса, там поясняется, что существует дохуйлион Вселенных со слегка отличными параметрами.

> и жизнь сама зародилась


Проблемы? Так-то уже воссоздавали из всякой первичнобульонной хероты органику, воздействуя электроразрядами и условиями, предположительно имевшими место быть 4 с гаком миллиарда лет назад на Земле.

> и расплодилась в миллиардах форм


Проблемы? За миллиарды лет-то?
Пикрил реакция Белоусова-Жаботинского демонстрирует циклическую (колебательную) химическую реакцию.
Помехоустойчивое кодирование, в частности, банальный дубляж (raid-1) генетической информации помогает регенерировать ей на протяжении миллиардов лет.
Мутации помогают созданию разнообразия биосистем.

> и ещё самоосозналась


А вот эта хуйня уже поинтересней будет. Ну, можешь учитаться Пенроузом, у него там интересные домыслы на этот счёт.

> Просто.


Ну, раз даже тыскозал, то так и есть.

> Откуда организмы научились смертоносный яд вырабатывать


Мутации же, и разные условия обитания. А потом экосистемы по какой-то причине начали пересекаться, и всё заверте... Что раньше было побочным продуктом (сракота из серных соединений, например), стало вдруг защитным механизмом. Потом означенные экосистемы застабилизировались дифференциальными уравнениями второго порядка, синхронизировали циклы взаимоуничтожения, поделили территорию, произвели самоселекцию наиболее приспособленных особей, снова решили чуточку помутировать...

>различать цвета,


Да, синтез дифференцирующих сенсоров ткани в единую систему -- такая неожиданность... за миллиарды лет... ни разу не было, и вот опять.

> вырабатывать и плести паутину узором


Ну, наверное, кто плёл её тонкой веревочкой просто умер с голоду и не оставил потомства, не?

> Просто всё как-то само по себе завертелось и существует.


Всё же, чую, это была самокритика.
31 448625
>>48604

>Блядь, как же у учёнодаунов всё просто.


Это такая самокритика или просто накипело от малолетних ЛеоТаксилистов?

> Вот не было нихуя


WUT? Прямо таки нихуя? Даже в вакууме запасена энергия, если что. А из математического нихуя даже самые отпетые Пенроузы физику выводить не осмеливаются.

> потом взрыв из нихуя


Ну а хули ты хотел, надо же как-то для детворы популизировать. Однако, нихуя, как уже указано выше, нихуя не нихуя, взрыв не то, чтобы совсем взрыв (взрыв пространства-то? для инфляционного периода? звучит забавно, ну ок, имплозия как взрыв имеет право на существование), да и сам процесс не то, чтобы сам процесс, ибо на начальных этапах времени как времени, в общем-то, вероятно и не было.

> и всё так само появилось


Проблемы? Энергия (хоть даже вакуума) не может перейти из одного состояния в другое?

> организовалось


Проблемы? Физические системы лично тебе обязаны быть изолированными друг от друга?

> откуда-то законы взялись


Если критика к конкретным энергетическим параметрам, то загляни в теории Мультиверса, там поясняется, что существует дохуйлион Вселенных со слегка отличными параметрами.

> и жизнь сама зародилась


Проблемы? Так-то уже воссоздавали из всякой первичнобульонной хероты органику, воздействуя электроразрядами и условиями, предположительно имевшими место быть 4 с гаком миллиарда лет назад на Земле.

> и расплодилась в миллиардах форм


Проблемы? За миллиарды лет-то?
Пикрил реакция Белоусова-Жаботинского демонстрирует циклическую (колебательную) химическую реакцию.
Помехоустойчивое кодирование, в частности, банальный дубляж (raid-1) генетической информации помогает регенерировать ей на протяжении миллиардов лет.
Мутации помогают созданию разнообразия биосистем.

> и ещё самоосозналась


А вот эта хуйня уже поинтересней будет. Ну, можешь учитаться Пенроузом, у него там интересные домыслы на этот счёт.

> Просто.


Ну, раз даже тыскозал, то так и есть.

> Откуда организмы научились смертоносный яд вырабатывать


Мутации же, и разные условия обитания. А потом экосистемы по какой-то причине начали пересекаться, и всё заверте... Что раньше было побочным продуктом (сракота из серных соединений, например), стало вдруг защитным механизмом. Потом означенные экосистемы застабилизировались дифференциальными уравнениями второго порядка, синхронизировали циклы взаимоуничтожения, поделили территорию, произвели самоселекцию наиболее приспособленных особей, снова решили чуточку помутировать...

>различать цвета,


Да, синтез дифференцирующих сенсоров ткани в единую систему -- такая неожиданность... за миллиарды лет... ни разу не было, и вот опять.

> вырабатывать и плести паутину узором


Ну, наверное, кто плёл её тонкой веревочкой просто умер с голоду и не оставил потомства, не?

> Просто всё как-то само по себе завертелось и существует.


Всё же, чую, это была самокритика.
32 448627
>>48625

>ибо на начальных этапах времени как времени, в общем-то, вероятно и не было.


Не могу представить энергию без времени. Помогите!
33 448628
>>48621
эволюция происходит в течении миллионов лет.
ничего "внезапно" там не бывает.

Та же проторазумная жизнь сотни раз зарождалась, но в итоге дальше не шла и вымирала. Ибо условий для дальнейшего развития просто не было. Либо тело в принципе не создано для дальнейшего развития, как у дельфинов, например, либо и вовсе деградировали так как в интеллекте для выживания смысла не было.

И что бы она пошла дальше звёзды должны были сойтись. Но так как это продолжалось в течении миллионов лет рано или поздно они должны были сойтись.
34 448630
>>48629 (Del)

>Две абстрактные величины, выдуманные людьми


А какие величины реальные?
35 448631
>>48627
пока можно мотать назад, то время есть. мы можем мотать в теории время и за пределы большого взрыва, так что оно и там должно было быть. Просто вселенная тогда была совсем в другом состоянии. А после перешла в другое состояние. Она и не обязательно была сжата в маленькую точку в момент большого взрыва. вполне возможно, что вселенная уже была огромной.
36 448632
>>48629 (Del)

>так как имеет даже правило нахождения численного значения


В правиле фигурирует время!
37 448633
>>48627

>Не могу представить энергию без времени. Помогите!


А разомкнутый электрический контур с батарейкой можешь?
Потенциалы есть, тока нет (утрирую. Утечками на разогрев проводника, саморазряд конденсатора, автоколебаниями и т.п. пренебрегаем).
Только вместо самого проводника -- пробой ("искра", "дуга") между "мирами". Це станет временем.
Вместо батарейки -- высокоэнергетичный вакуум. Це будет энергией.

Объяснил неправильно, но, надеюсь, понятно.
38 448636
>>48633
Спасибо, действительно представил! Но у меня есть большие сомнения относительно правильности такого представления...
39 448638
>>48634 (Del)
Сомнительная затея
sage 40 448639
>>48638
Смотря к каким.
41 448640
Могли бы пчелы или муравьи стать разумными, если бы они могли иметь большие размеры, что позволило бы развить нервную систему?
Возможен ли коллективный разум, были ли попытки его моделирования?
42 448641
>>48640
они не могут стать большими. и вряд ли бы стали разумными даже если бы выросли. труд там коллективный может, но это всё равно примитивные скрипты.
43 448642
>>48640

>Возможен ли коллективный разум, были ли попытки его моделирования?


Гугли: "Белковые игроки". Не совсем то, но близко.
44 448643
>>48641
А на планете с меньшей гравитацией?
Я имею в виду обретение разума не муравьем, а муравейником - возможно ли такое?
45 448644
>>48641

>они не могут стать большими


Но ведь они уже были большими когда-то.
46 448645
>>48642
Гугл выдает сайты про спортивное питание.
Или ты про это: https://ru.wikipedia.org/wiki/Foldit
47 448646
>>48643
Дело не в гравитации, а в том, что у них нет лёгких.
даже если у них сейчас мозг слегка вырастит, то они уже его питать не смогут.
Что уж говорить если тело в целом вырастит.

Ну а обретение разума муравейником это уже из научной фантастики. Возможность ничего так не наблюдается. Даже если они там будут активно передавать информацию через различные феромоны, то это всё равно очень медленно для разумной жизни и большая часть информации улетучится.

>>48644
Во времена когда кислорода было куда больше в атмосфере.
48 448647
>>48645
Да про это. На английском надо было гуглить наверное...
49 448651
>>48646

>Во времена когда кислорода было куда больше в атмосфере.


Это когда примерно, в конце мелового?
50 448659
>>48651
Я не эксперт по точным датам, но давно. Гугл даёт что 35% было в каменноугольном периоде. Видимо поэтому он так и назван, ибо из-за кучи кислорода в атмосфера образовывалось куча угля.
В конце мелового тоже выросло, но не так сильно.
51 448660
>>48659
Хотя не из-за кислорода уголь образовывался. А из-за того, что деревья сгнившие деревья особо в тот момент есть некому было. И углеород из них не превращался снова в СО2.
52 448671
>>48613
Это не бесконечная энергия, это штука, способствующая выделению энергии из концентрированной формы энергии-материи.
И потенциально штука, с помощью которой можно получать энергию из тёмной энергии.
53 448678
>>48550 (OP)
Я правильно понимаю, что если бы земля была на 50% больше, люди были бы на 50% ниже, или нет? Как вообще теоретически размер планеты влияет на жизнь?
54 448679
>>48678

> Как вообще теоретически размер планеты влияет на жизнь


никак, важна гравитация, а она хуй знает от чего зависит, пока щитается от плотности и состава
IMG20180922125525.jpg880 Кб, 2895x2341
55 448680
Привет научный. Короче есть один холодильник, который даёт импульсные помехи.
Я решил от них избавиться. Нашел схемку в интернете. А вот радиоэлементы точно такие как на схемке были - не нашел. Заказал с али что то похожее. Подскажите, всё ли нормально с номиналами? (Смотрите пикрелейтед). Я просто в этом нихуя не понимаю, делаю по аналогии. Ещё я варистор в схему сам заебашил, где то прочёл что так делают... Жду важи ответы.
56 448683
>>48613
Из свойств пространства-времени. А уж что там дальше это более сложный вопрос над которыми физики бьются десятки лет.

>Это ведь бесконечный источник энергии из ничего. Разве это возможно?


Невозможно и энергия не бесконечна и закон сохранения там так же работает. Что бы получить энергию от гравитации ты должен тело это подкинуть, после чего оно упадёт с той же силой, с которой ты её подкинул.
И даже не кидаешь, а бросаешь с горы, то ты всё равно должен будешь затратить столько же энергии, не считая передвижение своего тельца и прочего сколько и получится от броска.
57 448686
>>48683

>Что бы получить энергию от гравитации ты должен тело это подкинуть,


Ты не представляешь сколько всего подкинуло 13 с копейками миллиардов лет назад..
58 448687
>>48686
и сколько. вся энергия что выделилась во вселенной было в основном из-за термоядерных реакций. а при гравитационном схлопывании ничего не выделяется. Всё в ЧД остаётся
59 448691
>>48687

>а при гравитационном схлопывании ничего не выделяется. Всё в ЧД остаётся


42 это не только ответ на главный вопрос жизни, вселенной и всего такого.

Ктому же, не обязательно скидывать что-то именно на чёрную дыру.
60 448692
>>48691
Всё там нормальной с этой энергией при гравитации. Просто это хорошо описывается лишь ОТО, а в ту математику лезть как то не охота.
61 448695
>>48692
Ты о чм
62 448696
>>48695
*чём
63 448698
>>48695
О том, что никакой движок на гравитации ты не построишь.
64 448699
>>48698
Направляем на землю камень, ловим его платформой на зубчатой рейке с шестернёй. К шестерне прикручен генератор.
Очевидно что при падении на землю камня выделится больше энергии чем нужно чтобы его подтолкнуть к ней.

(Это если не хочешь собирать энергию от взрыва/удара напрямую)
65 448702
>>48699

>Направляем на землю камень


Прости, Господи, сразу не признал.
66 448703
>>48702
А ты что думал, жестокость?
Ничего личного, просто добыча энергии.
67 448705
>>48699

>Очевидно что при падении на землю камня выделится больше энергии чем нужно чтобы его подтолкнуть к ней.


нет. что бы свернуть большой камень с орбиты нужно приложить очень много энергии.

именно поэтому попасть в солнце на самом деле не тривиальная задача. ибо мы врашаемся вокруг неё с большой скоростью относительно, и этот импульс сохраняется и после взлёта ракеты. И для преодоления этого импульса нужно затратить не мало энергии.
68 448707
>>48705
Квазары светятся? Светятся.
Энергия выделятся? Выделяется.

Или скажешь что не светятся и не выделяется?
69 448708
>>48705

>именно поэтому попасть в солнце на самом деле не тривиальная задача


Олсо поясните за "устойчивые орбиты".
В моём понимании вращаешься чуть медленнее-падаешь на объект, чуть быстрее-улетаешь. С идеально нужной скоростью вращаться невозможно, вопрос только во времени до падения или улетания, которым иногда можно пренебречь и приравнять к бесконечности. Это и есть "устойчивые орбиты"?
Или реально есть какие-то устойчивые орбиты, и почему они есть?
70 448709
>>48707
Вещество падает в яму, если так можно сказать. Разгоняется и начинает излучать. Но что бы это вещество вытащить из гравитационного поля тебе потребуется потратить больше энергии.

Мы и сами используем энергию гравитации в некоторых местах, типа ГЭС. Но воду наверх после падения то поднимает солнечная энергия, что вызывает её испарение и последующее выпадение в осадках. Использовать гравитационную энергию можно, но наебать систему с помошью неё нет.

>>48708
Нет идеально нужной скорости. Врашаешься слишком медленно постепенно переходишь на более низкую орбиту и из-за этого разгоняешься и стабилизруешься на неё. врашаешься слишком быстро, наоборот переходить на более высокую орбиту и там стабилизируешься. Поэтому система сама себя балансирует.

Ну а вращение по орбите это и есть постоянное падение, просто всегда мимо падаешь. Только если опять же скорость слишком большая для этой орбиты, то начинаешь падать чуть дальше от цели, и наоборот.

Вон будет земля плоская, то луна бы давно на нас упала, а вот в круглую и маленькую ей просто сложно попасть. Летит то она по пораболе.
71 448739
Как думают местные господа, должен ли уметь отличник, выпускник-математик решить любую задачу школьного уровня? Наверное, ответят "нет", типа там слишком свои особенности, особенно у олимпиад...
А вот физик-бакалавр должен уметь решить любую школьную задачу по физике, даже олимпиадную? Все-таки, там нет всяких изъебов, как у математиков, все связано с реальностью.
72 448744
>>48739
Кому кто и что должен? Почему этот кто-то должен? Зачем ему это надо? Это какой-то тест? Мой опыт показывает, что умные люди умны во всем. Если такие легко решают любую задачу из универа, то и со школьными задачами они при наличии доступа к интернету легко справятся. Если учеба в универе дается тяжело, то и со школьными задачами есть вероятность не справиться, хотя и меньше чем у школьника.
73 448756
>>48739
должен уметь научиться. это отличает образованно от не образованного. первый обучаем, второй нет.
74 448776
>>48741 (Del)
физика и математика замкнутые системы в какой-то степени, но они еще и перемыкаются
75 448787
>>48550 (OP)
Как решать уравнения вида |f(x)|=g(x) ?
76 448788
>>48787
Разбиением на два, очевидно.
77 448791
>>48788
>>48789 (Del)
Целую в щёчку :3
78 448797
жэуниор Machine Learning инженер что то может дать науке в рф? могу работать за символические деньги.
79 448799
>>48789 (Del)
А если решать на поле комплексных чисел?
80 448800
>>48797
Можно сделать ИИ
81 448803
>>48683

>Из свойств пространства-времени.


но как это возможно? Почему это воздействие продуцируется само по себе и берётся из ниоткуда? Почему её нельзя обнаружить приборами?
Планеты бесконечно вращаются вокруг солнца, как и спутники вокруг земли. Почему нельзя из спутников добывать энергию?
82 448804
В каком состоянии находится вещество в центре земли?
83 448809
Если сны идут всего несколько секунд, значит наш мозг способен обрабатывать информацию со скоростью компьютера? Как вообще это возможно, что многочасовые события проигрываются всего за пару секунд, но это видится как фильм?
15373940929510 (1).png464 Кб, 612x408
84 448812
>>48803

> Почему её нельзя обнаружить приборами?


WUT?

>Планеты бесконечно вращаются вокруг солнца, как и спутники вокруг земли.


Да ладно? У меня от тебя распад протона гравитационная волна.

> Почему нельзя из спутников добывать энергию?


Кто сказал, что нельзя? Строй мега-башенки, изменяй центр масс, замедляй вращение. Разрешаю.

>>48804

>В каком состоянии находится вещество в центре земли?


В сжатом.

>>48809

>Если сны идут всего несколько секунд, значит наш мозг способен обрабатывать информацию со скоростью компьютера?


Если хуй эякулирует всего несколько секунд, значит няша вагина способна обрабатывать генетическую информацию со скоростью компьютера? Что сказать-то хотел?

> Как вообще это возможно, что многочасовые события проигрываются всего за пару секунд, но это видится как фильм?


Как вообще это возможно, что многочасовые события качества 240p кодируются DivX всего за пару секунд, но это видится как фильм?
85 448820
>>48809

> многочасовые события проигрываются всего за пару секунд, но это видится как фильм?


С чего ты это взял?
86 448822
>>48803
Добывая энергию гравитации из земли ты приведёшь лишь к тому, что она начнёт падать на солнце, а день будет всё длинее и длинее.
waterphasediagram2.gif51 Кб, 750x512
87 448827
Кипячу воду в чайнике на обычной плите в комнатных условиях. Вода закипает, значит температура воды - 100 градусов по Цельсию (или около того +/- 10 градусов, будем считать высота уровня моря). Попутно выкачиваю воздух из резервуара/окружения. Вода вскоре замерзает (плита всё ещё включена).
Вопрос: какая температура у воды? Я думал, что около 100 градусов по Цельсию, но по фазовой диаграмме у воды не существует такого состояния. Значит, температура кипения понизилась и вода охладилась (ведь она была 100 изначально)?
88 448835
>>48698
сколько готов заплатить за прототип?
89 448836
>>48827
С испарением из жидкости "убегают" самые быстрые молекулы, т.е. те самые горячие. Так что температура жидкости при испарении падает.
90 448838
>>48812

>В сжатом.


>


Там такое давление что ядра атомов должны соединиться.
91 448840
>>48838
Ты скозал?
92 448841
>>48840
Ну посчитай, если не веришь.
93 448842
>>48841
А твои где расчёты?
94 448845
>>48836
Спасибо, тоже видел такое объяснение. То есть температура падает с ~373K до, как минимум, 273K - тройной точки? (я просто смотрю на фазовую диаграмму, область "Solid"). Но картинка говорит нам, что это переход пар->лёд, т.е. десублимация? Но на подобных видео отчётливо (или не очень?) видно, что замерзает именно жидкость. Всё равно непонятно, пойду открою учебник.
95 448848
>>48835
что бы его в движок машины можно было засунуть и кататься на нём.
 .jpg145 Кб, 707x1024
96 448851
Анон, важный вопрос. Вот есть же всякие частицы которые испускают радиацию, гамма-лучи всякие. А вот можно из движения этих частиц энергию выделить? Ну там, ебать, молекулярные катушки из наноматериалов, а значит, гамма-кванты получается как меняющееся поле, ну и в молекулярных катушка эдс возникает. Можно, не?
Ну и если нет, то каким образом на молекулярном уровне извлекать энергию? Ну в смысле, не химическими реакциями, а чем-то еще.

Очень важный вопрос, подсоби, анночег.
97 448852
>>48827

> Вопрос: какая температура у воды?


0-200К. Судя по диаграме.
Она же моментально испариться, а потом момнетально замерзнет. Ну не моменталньо, а очень быстро.
98 448853
>>48852
>>48847 (Del)
Ясно, спасибо. Кину видео тем, кто проходил мимо и заинтересовался.
https://www.youtube.com/watch?v=glLPMXq6yc0
99 448856
Что есть логика в науке и вообще ? Вот в быту под логикой понимается самое рациональное поведение/выбор, а иногда самый выгодный. В науке, я так понял, логичность это детерминированность. Можно ли сказать что логичный это антоним вероятностный/стохастический ?
Ещё я вывел такую классификацию логик: формальная(которую можно описать строгими выражениями, формулами) и жизненная(семантическая, может её и можно формазилозвать, но пока не удалось никому). Формальная делится на разные виды, например: позиционная(propositional, это которую в универах проходят), на основе нейронной сети, контекстная(гугловский ворд ту век, то что анон полгода назад в треде про контекст описывал) и др. Позиционная делится на конкретные логики "под задачу": алгебра логики(даёт мат аппарат), логика предикатов(как применять мат. аппарат к утверждениям), высказываний(для более сложных утверждений), нечёткая(отойти от бинарности, чтобы градации были), релевантная(чтобы вывод делать только из действительно связанных вещей) и др. Как моя классификация ?
100 448858
>>48856
Логика это инструмент. Такой же как и математика, и другие.
Есть какие то известные моменты, из них логическим путём выводишь другие и после экспериментально их проверяешь.

Но логика не детерменирована, и логически объяснить одно явление можно сотней способов порой
101 448859
>>48851
Бумп
102 448860
>>48856

>В науке, я так понял, логичность это детерминированность.


Связь есть, но просто ставить одному слову в соответствие другое не совсем логически верно, даже если они и очень близки по смыслу, а не просто связаны каким-то образом, так обычно делается для простоты/быстроты передачи информации и подразумевает что на другом конце это смогут правильно интерпретировать.

>Можно ли сказать что логичный это антоним вероятностный/стохастический ?


Вероятностность, это такой метод скорее, но зачем вероятностность если ты например точно видел в отражении зеркала в какую руку твой друг монетку спрятал, может только для того чтобы определить не специально ли это было показано, чтобы потом обмануть.

>контекстная(гугловский ворд ту век, то что анон полгода назад в треде про контекст описывал


>Позиционная делится на конкретные логики "под задачу"


Ну любой раздел логики можно назвать методом под задачу, или тип задач, ещё кажется квантовая логика есть, но для неё очень много свободного времени надо.б
103 448862
>>48860

>>Можно ли сказать что логичный это антоним вероятностный/стохастический ?


Прочитал антоним как синоним.
Нет, ну конечно нет.
104 448863
>>48856
Просто почитай немножко про гносеологию и методологию науки, всё станет ясно. Логика и стохастичность не противопоставлены. Наиболее близкое, с оговорками - иррациональность и непоследовательность, а не стохастичность.
105 448865
А что, если две квантово-запутанные частицы - это одна частица?
106 448866
>>48851
"Фотосинтез". Всё уже придумано до нас. Хотя фотосинтез это немного не про ту длину волны. Еще можно Греть что-нибудь (рабочее тело) и преобразовывать энергию тепла в механическую.
107 448867
>>48865
А почему в двух разных местах тогда?
 .png282 Кб, 588x380
108 448869
>>48866

> "Фотосинтез"


> Еще можно Греть что-нибудь (рабочее тело) и преобразовывать энергию тепла в механическую.


Кпд маненький. Через генератор/ионизацию вроде как больше будет.

Если что нужно обосновать наномашины. Наномашины на уране, лол. Хочу сделать в игре магию, чтобы фаерболы кастовать, но нужно её как-то научно обосновать.

Можно еще через переходы, ну типо слои материалов, в которых возникает ионизация, что-то делать. Как вообще посчитать кол-во энергии? Как эти самые греи/зиверты первести в итоговые вольты и магнитыне поля?
Имеет ли вообще это смысл или излучение энергии несколькиз атомов урана слишком ничтожно?
109 448872
>>48867
А почему нет?
>>48871 (Del)
А, понятно.
Я ничего не понял
110 448873
Что местные аноны думают о следующих двух примерах.
Вот тут примеры про технический аппарат и слова английского языка - https://youtu.be/uiSIqSuDms8?t=1100
Вот тут пример со студентами - https://youtu.be/uiSIqSuDms8?t=1305
Я анон, котрый в прошлом треде спрашивал про энергию магнитного поля и намагничивание(который без тралинга спрашивал). Мне кажется есть люди, которые в состоянии не переоценивать свои знания(имхо я один из таких), но когда они начинают спрашивать, часто натыкаются на недопонимание и даже обвинение в траллинге(по личному опыту). Чаще просто остальным не интересно, вылезать из своего окопа мнимого понимания("зачем это надо если мы итак лучше других задачи решаем"). Конечно сюда часто залетают троли, но я призываю местных анонов более тщательно отделять их от обычных не понимающих с тупыми вопросами. ИРЛ я часто встречал догматизм и логическую реконструкцию(см. видео) того как должно работать, вместо объяснения как же работает на самом деле.Это вело к спорам и даже приходилось идти на провокацю, чтобы разжешь интерес в отвечающем. В итоге, он как бы учится вместе с тобой, отвечая на твой тупой вопрос.
111 448874
>>48869
А ты хорош...
112 448875
>>48873
Тут дело не в мнимом всезнании. Просто наука очень большая и в одну голову не влезает. Приходится что-то принимать как данность и не вникать глубоко.
113 448878
>>48875
Влазит. Я сегодня совсем рассеянный.
114 448879
>>48878
Влезает
115 448880
>>48874
Что ты имеешь ввиду? Если ты про ионизацию через бутерброд, то я взял вот этот https://patents.google.com/patent/RU2568958C1/ru шизопатент, пока искал добычу электрончиков через ионизацию. Там еще несколько есть, кстати.

И да, мне всё еще нужно как-то научно обосновать магию....
116 448881
Из-за чего возникает ветер?
117 448883
>>48875
Помоему ты не стал видос смотреть. Там с привязкой ко времени ссылки и тема видео не имеет отношения к моему вопросу, просто там два примера хороших попались в передаче этой.

>Приходится что-то принимать как данность и не вникать глубоко.


К такому случаю нет претензий, но смотри: вот ты чего-то не понял, нельзя же каждый раз сваливать ещё не понятое на то, что "это объясняется в более углубленном курсе" Вообще по личному опыту, такое чаще было чем просто непонимание, но не всегда мне даже говорили об этом. Вспомнил: в школе мне не объяснили норм закон рад распада, т. к. он требует распределения Бернулли, а ТеорВера в школе нет. Мысля школьными понятиями я задавал вопросы "а что если атомов нечётное, сколько распадётся ?" "а если 1 атом не может же половина стать через полупериод ?" С такой постановкой вопроса меня слали сам понимаешь куда. Я просто хотел отметить, что не все с идиотскими вопросами трали.
118 448884
>>48880
Нет, с бутербродом всё в порядке. Я про то, что ты вначале произвел впечатление человека, который действительно понятия не имеет о том, как решить данную проблему, но потом продемонстрировал большую подготовленность в данном вопросе. Я желая помочь тебе чем могу сел в лужу и выставил себя ослом. Возникло подозрение, возможно неверное, что ты просто тролишь.
119 448886
>>48885 (Del)

>Странно, что ты сам не мог дойти до Бернулли распределения, всё же теорвер в школе малюсенький но есть.


Не во всех школах. Не все люди такие сообразительные.
120 448887
>>48884

> Нет, с бутербродом всё в порядке.


Уверен? Мне вот кажется что это шизопатент, тупо понапихано "титан, золото, ну всякое-там такое", будто на бум писали.

> но потом продемонстрировал большую подготовленность в данном вопросе


Нет, лол, я даж не знаю какое кол-во энергии производит уран через ионизацию. Хотя нормальный челик бы на раз два ответил...

> сел в лужу и выставил себя ослом


C чего вдруг? Просто я с самого начала не правильно высказался и ты не понял меня.
121 448888
>>48883
Думаю, что тут дело еще и в том, что для некоторых людей вещи вроде распределения Бернулли являются простыми и интуитивно понятными, а кому-то приходится приложить усилия, чтобы вникнуть. Прошу прощение за образность, но "понимание" одаренных людей подобно широкому лучу прожектора, который захватывает не только то на что направлен, но и некоторое количество расположенных рядом объектов, а понимание менее умных подобно узкому коллимированному лучу который выхватывает только непосредственно объект изучения.
122 448891
>>48890 (Del)
Это бесспорно!
123 448892
>>48890 (Del)
Я своим просто вожу из стороны в сторону. )))
124 448893
>>48881

>Из-за чего возникает ветер?


В гугле нашёл, оказывается из-за БВ, я не троллю
125 448894
>>48893

>БВ


Большого взрыва?
126 448895
>>48871 (Del)
Они через поле как-то взаимодействуют что ли?
>>48867

>А почему нет?


Ну это было странно, если идти от классического понятия частицы, по крайней мере, никогда о таком не слышал, и сам не видел и не могу представить, что бы частицы так вдруг себя повели.
127 448897
>>48894
Ага, ну и там дальше по цепочке..
>>48895

>и сам не видел и не могу представит


Ну это было бы не странно, конечно, но, чтобы что-то могло быть в двух местах физически, оно что, должно раздвоится, или что? Кхм, вот это кажется странным.
128 448900
>>48897

>могло быть в двух местах физически


Нет, не так, понятно что физически, с этим постоянно путаница, но ещё и одновременно, если бы они через поле как-то взаимодействовали, это еще понятней было бы, но никто пока видимо не знает точно каким это образом происходит, или точно это могут знать и объяснить на пальцах всего человек 20-30.
 .gif48 Кб, 454x350
129 448901
>>48851
>>48869
И еще один бумп, поцаны.
130 448904
>>48822
Почему? Всего лишь генератор из неё построим. Никто так и не объяснил что это такое, значит никто не понимает. Бесконечная энергия, которая берётся неизвестно откуда и вечно воздействует на всё вокруг.
131 448907
>>48904
Потому что вечный двигатель невозможен.
132 448908
>>48907
Я про гравитацию, а не вечный двигатель. Все морщатся при упоминании экстрасенсорики, магии, телепатии, но вот вам ничем не объяснимая неисчерпаемая энергия между предметами ими же излучаемая и все в неё верят.
133 448909
>>48908

> Я про гравитацию, а не вечный двигатель


Ты как раз предлагаешь построить вечный двигатель на гравитации.

> Все морщатся при упоминании экстрасенсорики, магии, телепатии


Потому что это давно обоссаные гипотезы без экспериментального подтверждения.

> но вот вам ничем не объяснимая неисчерпаемая энергия между предметами ими же излучаемая и все в неё верят.


> ничем не объяснимая


Есть такое

> неисчерпаемая


Нет, энергии конечное количество

> излучаемая


Нет, от Солнца Земле с помощью гравитации энергии не передается. На поддержание гравитационного поля энергия тоже не расходуется, иначе объекты теряли бы массу.
134 448919
>>48908
Что такое гравитация? Искажение пространства-времени под влиянием массы. А откуда взялась масса? Из энергии при большом взрыве.
Так что гравитация это не какой особый источник энергии. Вся энергия, которую ты можешь выжать из гравитационного поля, в нём так же была заложенна во время большого взрыва по своей сути. И добывая энергию из гравитации активно ты либо должен "поле" разглаживать, а значит переводить массу в энергию, что не совсем гравитационная энергия, конечно, но в этом поле как бы и спрятана. Либо наоборот - дальше скручивать массу в один большой комок, дабы добывать эту энергию при падении в более глубокую гравитационную яму.
135 448921
>>48919
Ну и приливные силы я ещё забыл. Но и там скорость вращения земли из-за них медленно, но замедляется. Хотя в такой громадине вращающейс и запасенно огромное колличество энергии и остановить её не так и просто. А если начать вытягивать приливную энергию из куда более тяжёлые и быстро вращающихся объектов, то там и вправду её можно выжать очень много. Но это уж из области научной фантастики, ибо надо летать к другим звёздам, желательно нейтронным и как то выживать рядом с ними.

Так что из гравитации земли можно вытянуть энергию если только начать её замедлять. Что не круто. поэтому и остаётся два варианта - просто падение, как при ГЭС, тут солнце полсе воду поднимает само. Ну или превращение массы в энергию, что отчасти тоже пока из области научной фантастики. Ибо при попытках аннигиляции на создание антиматерии уйдёт в разы больше энергии, чем после выжмешь.
136 448924
>>48858
>>48860
>>48863
Я чисто с бытовой позиции рассуждал. Скорее даже играл со словами, как-то задумался о синониме к слову не логичный. Вот поведение животных и, ну скажем шизофреников, не логично, т. к. оно не детерминировано. Кстати еть слово для недетерминированного и не стохастического оновременно ? Помоему так не бывает. С другой стороны рандомное поведение тоже может быть логичным в некоторых слуачях.
Если у ситуации было 3 исхода 80 % на 1 и по 10 на остальные, логично выбрать 1, а допустим, постфактум мы обнаружили, что на самом деле имел место третий и это тоже будет логично(в плане что произошёл 1 из 3, а не четвёртый, несуществующий вариант) и здесь нет противоречий.
137 448926
>>48919
У меня от этого поста гравитация, имеющая отрицательную энергию, ВНЕЗАПНО отделилась.
Тут подумалось мне, что из гравитации, энергия которой отрицательна,
можно выжать как-то и антиматерию, ведь E=mc^2, m = E/c^2, а при -E = -mc^2, -E/c^2 = -m;
а потом, эту антиматерию взять и аннигилировать с материей в охуенно энергичные джеты.

Но что если пространственно-временной континуум, обладает массой на астрономических расстояниях,
и с учётом того, что гравитация имеет отрицательную энергию, и масса эта типа ещё и отрицательна,
то тут уже как-бы поясняется - Барионная асимметрия Вселенной...
Ну типа энергия антиматерии ушла в пространство и гравитацию в момент БВ,
потому гравитация - отделилась, в результате - осталась материя.
Так вот, если так, то можно ли было бы как-то, избирательно,
аннигилировать с материей - само пространство, и антимассу внутри него скрытую,
снижая при этом количество материи и уровень гравитации во Вселенной?

Что-то вроде управляемого локального сингуляризирования пространства,
с направленным испарением чёрных дыр в виде джетов.
Представляю себе установку, в которой исчезает материя, по мере того, как в это устройство,
направленно падает внезапно - само пространство, и исчезает тоже там,
а из устройства этого - энергичный поток гамма-лучей прёт прямо на солнечные элементы.
138 448927
>>48924

>Если у ситуации было 3 исхода 80 % на 1 и по 10 на остальные, логично выбрать 1


Что логичнее выбрать это уж теориявероятностей. Ибо взять банальных пример из ставок, когда у тебя за 80% коэф 1 к 1.1, а на 10% 1 к 100, то куда логинчее ставить на второе.
В первом случае заработаешь с доллара 10.81.1=0.88, то есть ставка минусовая.
Во втором 10.1100=10. То есть сильный плюс. Ошибка же тут может быть если ты только шансы неверно оценил.
139 448928
>>48926
Отрицательная энергия в гравитации это просто законсервированная энергия, к тому же в ОТО в добавок существуют гравитационные волны, а уже положительная энергия в гравитационном поле.
140 448929
>>48927
10.81.1
Знак зведы что ли не пашет?
141 448930
>>48929
10,81,1
142 448931
>>48927
Я имелл ввиду, когда нельзя ввести функцию оценки результата, нам просто предлагают угадать и вышло так, что 3 исхода из них один - 80 % и 2 по 10 %. Угадываем - молодцы. А выигрышей тут нет, всё одинаково.
143 448932
>>48924
Тебе понятие иррациональности вообще знакомо?
144 448933
>>48932
Да, точно, это то что нужно, спс.
145 448936
Почему в темноте я вижу не полную черноту, а темнотемно-багровую муть и как будто в мелких пикселях, которые разноцветно бурлят, как пена? Это вообще нормально, просто нервный шум? И что видят ослепшие, полную черноту-таки?
146 448940
>>48924

>Вот поведение животных и, ну скажем шизофреников, не логично, т. к. оно не детерминировано.


оно детермировано нестандартными с т.з. здорового человека функциями мозга, почему например по полнолуниям, и у тех и у других наблюдаются обострения. а потом идут на спад? возможно шизофреническое поведение ближе к животному, и с этой стороны его можно считать даже более стандартным/детерминированным.
147 448942
>>48940
Причём тут ближе к животному. Мозг это система. Если в этой системе появится ошибка, то логично, что начинаются глюки и неадекватные решения.
148 448943
>>48942
Это означает что ошибка детерминирована др. факторами, в общем в контексте вопроса это не совсем подходящее понятие.
149 448944
>>48885 (Del)

>Странно, что ты сам не мог дойти до Бернулли распределения, всё же теорвер в школе малюсенький но есть.


Я в 2011 году школу кончил. В ЕГЭ теорВера не было вот и не учили, хотя в учебнике он был да. На физике чётко говорят, что за период полураспада распадается ровно половина атомов(спокух, я давно уже знаю, что это не так). До сих пор я когда на эту тему с людьми говорю, они думают что я определения не смог выучить и нибось ни одной задачи решить не мог, а значит верить мне не стоит.
>>48875
>>48888
>>48885 (Del)
Я всё же предлагаю посмотреть видос, особенно вторую ссылку отсюда >>48873 . А как местные одарённые аноны становятся уверены, что понимают "истину как она сегодня изветстна", а не логическую реконструкцию как должно быть по идее ? Я вот начинаю придумывать себе конкретные тесты, пытаюсь отследить что куда перешло, переместилось, нет ли элементарных проблем в математике типа деления на 0, нарушения тождеств, полтора землекопа и т. п. Иногда я не могу сам ответить себе наэти вопросы, тогда я гуглю, если пусто то, пощу их в этот тред.
150 448945
>>48943
Детерменированность детерменированостью, но всё это настолько сложные системы, что даже и не надейся их просчитать.
151 448974
Там говорят, что гипотезу Римана доказали. Пиздят или нет?
152 448981
Так что там с квантами? Одна частица или лве при квантовой запутанности?
15377184604130.jpg43 Кб, 477x693
153 448982
>>48981

> создаёт полость внутри суперпрочного цельнолитого шара и делит оный на два телесных сектора, засовывает в центральную часть (в исходную полость) антивещество, взрывает оное


> импульсы телесных секторов равны по модулю, противоположны по направлению


> спрашивает на дваче, на сколько частей был разбит шар


> ждёт, пока его обложат хуями

154 448984
>>48962 (Del)
Минопроходил
Ходит байка, что Максвел вывел свои уравнения из теории эфира. Что на это скажешь?
155 448989
>>48981
Ответов тебе дано предостаточно, ты их понять не можешь в силу незнания теории. Не нужно учить матан 10 лет подряд, чтобы понимать, что такое суперпозиция, пространство Фока, или неприводимые унитарные представления группы Пуанкаре. Но нет, лучше дрочит в дотку, постить смехуечки на двоче, бровзить паблики вк и кичиться своим отменным знанием первых абзацев статей Википедии.
156 448990
Какой предполагаемый размер вселенной(полный), согласно модели большого взрыва и инфляции?
Раз 5 уже находил видосик где это вскользь упоминается и каждый раз забываю а потом долго не могу найти.

Почему в некоторых видосиках, которые не опровергают теорию большого взрыва, говорят что вселенная бесконечна? Взаимоисключающие же вещи.
157 448992
>>48990
Ещё один профессиональный смотритель ютюба. Современная (а не из 1960х) теория большого взрыва ничего не предполагает о конечности или бесконечности вселенной.
158 448993
>>48992
Не выёбывайся, няшка-хуяшка.

>Современная (а не из 1960х) теория большого взрыва ничего не предполагае о конечности или бесконечности вселенной.


Почему? Если она предполагает наличие начала, значит должна и предполагать наличие конца. Если не должна, то почему?
159 448995
>>48994 (Del)
Да, я и так знал что лучше быть профессиональным смотретелем ютуба, чем профессиональным смотретелем выступлений степоненко и петросяна, без твоих ответов.
160 448997
>>48996 (Del)
Как бы почитай название треда.
161 448998
>>48995
Это не мой коммент. В это, возможно, трудно поверить, но в сай сидит более одного человека, которые в состоянии осилить основы функционального анализа, теории представлений, и КТП.

>>48993

>Не выёбывайся, няшка-хуяшка.


Модельный ответ. Действительно, дальше видео на ютюбе тебе с таким отношением не продвинуться. Заметь, к слову, что я на вопрос ответил (тут >>48992), и моё обращение не было унижительным или оскорбительным - с научпопом нужно быть настороже, на что я и указал (и что ты и доказал).

Лучше быть эджи пацаном, обзывающем всех "няшкой-хуяшкой" (тебе что, 14 лет?), чем попытаться разобраться в теме.

>>48997
Не готов понять или принять ответ - не задавай глупых вопросов.
162 448999
>>48996 (Del)
Я могу в логику в отличи от тебя(на основании твоего ответа про моркову и апельсин).
Было начало-время когда размер был конечным и маленьким. С этого момента прошёл конечный отрезок времени, известный. Есть теории и модели, описывающие скорость расширение вселенной в разные периоды этого конечного отрезка времени, сначала сверхбыстрого-инфляции, затем относительно медленного, обычного, но ускоряющегося.
С учётом всего этого должен быть конечный размер, который должны определять эти теории и модели.

Хотя сейчас вспомнил другой видосик, в котором говорилось о том что расширялась наблюдаемая часть вселенной, т.е. целиком область стремящаяся к бесконечной плотности могла быть бесконечно большой, а в стремящуюся к бесконечно малой точке была сжата только наблюдаемая часть вселенной. Согласно этому получается бесконечный размер с бесконечной плотностью начал расширяться в бесконечность бОльшего порядка, а плотность стала уменьшаться, стремясь от бесконечно большой величины к бесконечно малой..
163 449001
>>48999

>С учётом всего этого должен быть конечный размер, который должны определять эти теории и модели.


Не должен. Ибо бесконечность никак расширению не противоречит.
164 449003
>>48999
Современная теория большого взрыва ничего дополнительно не говорит и не требует о размере вселенной. Импликации следующие:
1) Если сейчас вселенная бесконечна, то она и была бесконечна всегда.
2) Если вселенная была кончена, то она конечна и сейчас.
Ничего больше теория не говорит.
165 449004
>>49003
очевидный фикс конечна
166 449005
>>49002 (Del)

>Но тогда зачем ты вопрос задавал?


>>48990

>и каждый раз забываю а потом долго не могу найти.


Потому что я забыл же.

А то что было во второй части того сообщения, что имелось ввиду в том видосике, вспомнил и понял только сейчас, пока набирал первую часть. Слишком там витьевато и образно было, как красивый слишком поверхностный рассказик, без конкретики и остановок "почему", просто бегло факты не совсем в прямой форме-полуобразно, чтобы было "красивее".
15345405118050.jpg211 Кб, 736x736
167 449006
>>48636
Оно педагогически неверно.
Это как объяснять первоклашке азы электростатики через представление об электричестве как о жидкости в трубе. Вроде и можно, и даже почти достоверно. Но потом у подопытного начинается каша в голове, скорость движения электронов (не поля!) оказывается у него близкой к световой (разумеется, он не петавольтами оперирует), магнитная компонента у него отождествляется едва-ли не с диффундированием, о переменном токе, скин-эффекте и прочих вихрях Абрикосова лучше промолчать, потому что они в такой треш превращаются, что фильмы Михалкова отдыхают.
Также и здесь. Сходу создаётся впечатление, например, что существует некоторое "правремя", в течение которого и происходит флуктуация ("пробой"). Это, скорее всего (хоть и не обязательно), неверно.
И остальные "выводы" новообращённого могут оказаться столь же ошибочными.
Поэтому педагогически правильно посылать таких вопрошающих нахуй. Ну или за учебник квантовой хромодинамики, что одно и то же.
168 449009
если смазкой можно смазывать железные сплавы, то можно и медные?
так как медные устойчивее к окислению, чем железные. верно?
169 449010
>>49009
Под окислением обычно понимают окисление кислородом, а в состав смазкок могут входить присадки разной степени активности, которые могут вступать в химические реакции с разными материалами.
Но в общем случае ничего плохого не должно случиться, хотя есть такие масла и смазки(точно помню что читал про масло, вроде бы редукторное, которое запрещено использовать в редукторах с цветметом внутри, и это каким-то допуском оговаривается, если оч надо могу попытаться вспомнить/найти) которые нельзя использовать с цветметом.
170 449013
>>49010
>>49009

>Масло с допуском ГЛ-5 Агрессивно по отношению к цветным металлам.


>Масла API GL-5 обладают лучшими противоизносными свойствами, чем GL-4, однако, в силу повышенной агрессивности к цветным металлам (синхронизаторы, втулки подшипников скольжения и т.п.), их не рекомендуется применять в коробках передач, для которых рекомендованы масла API GL-4.


>API GL 5 — применяется в высоконагруженных гипоидных передачах, а так же в некоторых КПП в которых конкретно указано применение масел именно этого класса. В этом классе масел максимально возможная концентрация присадок содержащих серо-фосфор, максимальные противозадирные и противоизносные свойства.


>Связано это прежде всего с тем, что классы трансмиссионных масел основаны на фактическом количестве серо-фосфор содержащих противозадирных и противоизносных присадок, которые агрессивны к цветным металлам.

171 449017
test
172 449018
>>48995
Проблема большинства популярно научных видео на ютубе в том, что они берут последние научные новости и выдают их чуть не за истину. И то это ещё среди тех, кто в эту истину пытается.
Как я помню в одном таком утверждали, что наша система аномалия, ибо в большинстве найденных Гиганты находились у звёзд, когда у нас это иначе. Но объяснение тут куда проще. Просто такие планеты проще всего находить. А на деле такие системы наоборот редкость.
173 449019
>>49018

>Но объяснение тут куда проще. Просто такие планеты проще всего находить. А на деле такие системы наоборот редкость.


Этакий survivorship bias, про который местные двощеры тоже никогда не слышали.
174 449021
Никак не могу понять что такое "плезиоморфный" и "апоморфный", на википедии написано неясно а в других местах ссылки на вики.

Вот:

>Апоморфный признак (или апоморфия) — это характеристический признак, выделяющий группу видов среди множества других и унаследованный членами группы от их общего предка. Плезиоморфный признак (или плезиоморфия) — это характеристический признак, унаследованный видами группы видов от их общего предка и характеризующий большую по объёму группу, содержащую первую в качестве своей подгруппы[4]. Прилагательные «плезиоморфный» и «апоморфный» используются вместо «примитивный» и «продвинутый» для предотвращения оценочного суждения об эволюционных формах, поскольку оба они могут обозначать структуры, крайне полезные при некоторых обстоятельствах и вредные — при других.



Как это понимать? Я несколько раз прочитал текст и даже не могу сказать что мне в нем непонятно. Просто не могу представить и все. Я тупой или текст?
175 449022
>>49019
Мамкин комнатный умник, что-то ты сегодня разошёлся.
176 449024
>>49022

>на дваче сидит один человек


Умник тут точно не ты.
177 449030
Как запасти большое количество электроэнергии для домашнего хозяйства как минимум на пару лет?
Была идея закупить генератор и солярку, но оказалось, что свойства солярки со временем меняются и она становится непригодной для использования. Есть способ законсервировать солярку? Или нужно искать другой способ запасаться энергии?
178 449033
>>48936
Собственный шум приемной системы.
179 449035
>>49034 (Del)
Да. Всё оказалось просто. Не думал, что пропан-бутан можно хранить неопределённое количество времени. Думал, что тоже как-то портятся.
180 449036
>>48936

> Это вообще нормально, просто нервный шум?


По сравнению с тем, что приходится фильтровать твоей слуховой коре - это хуйня.
Lolishit - Misao Glitch Error [Chest Flattener] {HD}.mp43 Мб, mp4,
576x360, 1:37
181 449046
Поясните физику стенопа, почему он работает, как линза?
182 449066
Почему теория бв идеалистическая хуита?
183 449068
как развить эмоционально волевой фактор?

интернет как правило его съедает
184 449069
Чувствую себя идиотом, помогите разобраться с электромагнитным излучением. Что физически собой представляет энергия фотона? Если скорость и масса постоянны, то откуда разные длины волны (интернет говорит что от длины все и зависит, но это же следствие?). Единственное что могу представить это "движения" на квантовом уровне, но получается как-то мутно.
185 449070
>>49069
Массы у фотона нет. Есть только энергия и импульс. А по поводу, почему частота так связана с энергией... лучше не лезть сюда, и просто пока запостулировать себе это.
15379708147350.jpg51 Кб, 650x427
186 449081
>>49066

>Почему теория бв идеалистическая хуита?


Ты про Божественное Вмешательство? Ну, не сказать, чтобы прямо хуета, тут или антропный принцип на теории Гамова / Гутова (ну, в последнем случае ещё хоть как-то), или Смолинова "генетика" чёрных дыр, или Божественное Вмешательство, или я не знаю. На фоне представленных кандидатов, БВ выглядит вполне сносно.
Почему бы высокоразвитому, осознающему себя сверхсуществу не компактифицировать вакуум так, чтобы образовавшиеся в нём существа не были разделены на два пола, чтобы описывать свои проблемы на двачике (явленном этим недоинфузориям интерфейсе означенного сверхсущества)?
187 449082
Не то чтобы вопрос по науке, но может кто подскажет. Нужно из одной фазы в розетке сделать 3. Преподаватель сказал про частотный вращатель но я про него что-то не могу инфу найти. Частотный преобразователь не подходит, должно быть что-то проще.
15379894423040.png40 Кб, 512x512
188 449083
>>49068

>как развить эмоционально волевой фактор?


Берёшь свои яички, кладёшь их промеж электродами на напряжение в 40 вольт.
Берёшь оператора, который даёт тебе задания и оценивает их выполнение. Если твоя оценка не ахти, разряд.
Чтобы не было возможно снять, гугли всякие там трусы верности. Ионистор внутри оных, без возможности изъять или "коротнуть".
Чтобы оператор не охуевал, нужно взять в заложники его попугая (передаёшь птицу неизвестным тебе третьим лицам, которые после (электротерапии), за биткоины, в означенном месте и означенное время проведут твою биоидентификацию и отестируют с помощью полиграфа. Если ты доволен своей волей, то попугай возвращается владельцу). Если в заложники взят не попугай, ничего не сработает, запомни это.

Савельев рекомендует, спецслужбы рекомендуют. Только попугай против.
189 449084
>>49066

>Почему теория бв идеалистическая хуита?


Ты скозал?
190 449085
>>49046
бамп
191 449086
>>49083
ха, в китае кстати так и делают, в исправительный лагерь отправляют,и фигачат шокером зависимых, я бы не отказался кстати
15345493878383.jpg11 Кб, 150x127
192 449087
>>49082

>Нужно из одной фазы в розетке сделать 3


В смысле, расщепитель фаз тебе? Или что?
Так-то подключаешь треугольником, на одной из сторон конденсатор для сдвига фаз и всё. А вообще, иди-ка в какой-нибудь /ra с таким не иди.

>>49084
Цык, я первым его обоссал!
15346033755500.jpg63 Кб, 620x620
193 449089
>>49086

> я бы не отказался кстати


Ты бы и от попугая не отказался.
194 449090
>>49087

>расщепитель фаз


Вот хз, возможно его и имел в виду.
Попробую туда сходить, в любом случае спасибо.
268zj7.jpg25 Кб, 645x729
195 449091
196 449092
>>49082
Берешь подключаешь к одной фазе движок, им крутишь генератор на 3 фазы.
197 449094
>>49081
Что за шизоидный бред ты выдал?
198 449095
>>49094

>Что за шизоидный бред ты выдал?


А вот и наш школьник подъехал.
199 449097
>>49046
Он работает по принципу камеры обскура.
200 449105
А я опять врываюсь в этот итт тред с вопросом о возможном полном размере вселенной.

Есть ли какие-то теори, в которых пытаются предположить возможный полный размер вселенной, основываясь на том, что видно в/по видимой части вселенной?

Например что-то вроде "если кривизна пространства на видимой части вселенной 0.0000000....°, а оно целиком должно быть как шарик, то полный размер вселенной= 360/0.0000000....° * "размер наблюдаемой вселенной"
или типа того, я хз
201 449112
Верно ли утверждение, что электрический заряд частицы - фундаментальное свойство? Если нет, то чем он обусловлен?
202 449113
>>49105
теорий может быть хоть 10000, но нормальных среди них точно нет, которые можно взять и как то проверить хорошо.
203 449120
>>49112
Верно.
204 449125
>>49112
Нет. Заряд это не свойство частицы, научно невозможно определить свойства чего либо, можно только посмотреть как нечто взаимодействует с чем-то, это всё, что доступно эксперименту, "свойства" же являются своего рода дохами, которые учёные вселяют в вещи, чтобы объяснить почему они взаимодействуют друг с другом так, а не иначе.
205 449126
>>49125
духами*
206 449128
Давеча услышал об осознанных снах. Думал антинаучная хуйня. Вообщем долго распинаться не буду - я обалдел. Это работает, просто ахуел. Это самое удивительное в моей жизни . Мне просто интересно, почему это не распространено широко. Грубо говоря, для социофобов это прекрасная возможность.
207 449129
>>48550 (OP)
Правда, что звёзд в млечном пути больше, чем атомов во вселенной?
208 449130
>>49128
Потому, что это наркоманИя.
209 449131
>>49112
Не совсем.
Фундаментальное свойства это только набор некоторых параметров поля, которое уже порождает частицы.
Сам заряд это результат взаимодействие нескольких полей. В случае электрона - спинорного и электромагнитного. В этом случае заряд это внешняя видимая характеристика, да еще благодаря симметрии строго сохраняется, а из-за квантовых эффектов дискретная величина.
Впрочем в некоторых случаях заряд частицы может изменяться, даже стать дробный, и это уже рассматривают как результат изменение коэффициента взаимодействия между полями.
210 449132
>>49046
Ну хуй его знает, сделай задачу для школьников: постой ход лучей через щель от дальнего объекта на экран - и ты познаешь истину!
211 449133
212 449135
>>49112
Не знаю, почему другие аноны не написали про две интересные вещи - то ли сами не знают, то ли тебя не захотели пугать (то ли считают, что это не релевантно).
Могу позже подробней написать, вот вкраце:
1. Заряд интересен тем, что он сохраняется. Это следствие теоремы Нётер, а именно глобальной калибровочной U(1) инвариантности волновой функции. То есть то, что волновую функцию можно умножить на произвольную фазу, влечёт за собой сохранение некой величины (которая совпадает с эл. зарядом)
2. Если мы потребуем локальной калибровочной инвариантности теории, то можно вывести, что кроме поля частицы должно существовать ещё некое векторное поле, обеспечивающее инвариантность лагранжиана. Это поле, оказывается, и есть электромагнитное поле.

Симметрия рулит, парни.
1401539267232827687.jpg63 Кб, 800x578
213 449139
Почему учёные так легко поверили Эйнштейну? По нему же видно, что он жид пархатый.
214 449141
>>49139
А много учёных не жидов?
или ты имел ввиду как это жиды поверили другому жиду, лол
215 449144
>>49139
Проверили и поверили. В науке всё именно так и работает, кто эксперимент предсказывает, тот и молодец.
216 449146
>>49139
Жидом он был по роду. А по крови русским. Жиды же похищали детей.
217 449148
>>49097
>>49132

Два дебила, спрашивают: "как конкретно работает?", отвечают типа "Работает нормально".
218 449149
>>49141
Ашкеназы не евреи.
15346181324880.jpg91 Кб, 822x720
219 449158
>>49128

>Мне просто интересно, почему это не распространено широко.


Не могут заставить себя даже осознаться.
Ну или какие-ещё индивидуальные нюансы.
Например, я просто дох со скуки, когда практиковал.
А вот когда вернулись няшные кошмарики и лабиринтики с логическими ловушечками, всё стало зашибись обратно.

Я тебе секрет открою. Который реально действует, по меньшей мере, для меня. Уже далеко после того, как случайно попробовал и вошёл во вкус, узнал, что Кастанеда такую же технику использовал.
Надо банально во сне лечь (с целью заснуть во сне) ровно в ту самую позу, в которой ты сейчас спишь. Если позы совпадают, заснуть во сне таки удалось и тебя резко не вышибло, прозревай столь же реалистичный мир, как и до (первого, реального которое) засыпания, но только с незначительными отличиями (по которым ты можешь понять, что это всё же сон). Интенсивность ощущений местами выше реальной, также имеет место их полнота (запахи, вкусы, вестибулярка).
Чтобы выйти, ляг ещё раз в ту же позу, проснёшься во сне первого уровня. Оттуда просыпаться ты и так умеешь, если что.
Есть ещё некоторые любопытные нюансы, но я не хотел бы их палить, так как рассчитываю на независимый эксперимент. Если добьёшься успеха в упомянутом мной деле, отпишись сюда (на тех наркоманских досках не сижу) о достигнутых результатах, сравню, упорядочу, выложу сюда.
15378233643460.jpg22 Кб, 398x510
220 449161
>>49158
Примечания-дополнения.
Примечание 1. Если поза относительно точно не совпала, ничего страшного, просто нихера не выйдет, будете ощущать себя запутанного (одеялом?) с визуальной серизной вокруг. Сновидение нестабильно, само пытается вас "выдавить" или в явь, или в сон первого уровня. Аналогичного эффекта можно добиться, если во сне постараться войти в своё отражение в зеркале.
Примечание 2. Первые попытки могут не увенчаться успехом, более того, вас могут преследовать резкие псевдоудары (как от грузовика), в результате коих можно проснуться с повышенным сердцебиением. Ничего страшного
Примечание 3. А вот умирать во вложенном сновидении не советую, т.к. сам не пробовал. Предполагаю, что что на основе проприоцепторного успокоение человек входит в состояние гипнотического транса, если при этом его что-то "убьёт" в его воображении, он может (вероятно) и не умрёт, но невроз схватит наверняка. Так что осторожней там.
221 449164
Почему я не могу понимать науку? Для меня необъяснимо почему вселенная не бесконечна, но постепенно куда-то расширяется, а число пи имеет бесконечное значение после запятой. Почему предмет может бесконечно уменьшаться, но расстояние не может и приходит к нулю, а затем переходит в минусовое значение. Почему парадоксы могут спокойно существовать нарушая законы логики.
15336753560083.jpg410 Кб, 976x1815
222 449165
>>49164

>Почему я не могу понимать науку?


Потому что ты не во времена СССР, когда в просвещение молодёжи вкладывали рублосы. С другой стороны, у тебя есть интернет. Правда, на смартфоне куда слаще в какой-нибудь майнкрафт загеймать, да? Хуй тебя знает, почему. Может ты гуманитарий.

> Для меня необъяснимо почему вселенная не бесконечна


Для учёных тоже. Мне неизвестны достоверные научные факты, которые бы утверждали её конечность.

> но постепенно куда-то расширяется,


Учёные тоже в ахуе. Они то в лучшем случае, выясняют, каким образом, о почему заикаются редко.

> а число пи имеет бесконечное значение после запятой


Лол.
В системе счисления по основанию пи - вполне себе конечное, например. Я тебе его точно могу привести: "10".
Потому что нет конечного номера шага, на котором ты, обкладывая площадь круга квадратиками, преуспеешь в этом.

> Почему предмет может бесконечно уменьшаться


А посоны-физики-то и не знают! Всё какой-то длиной Планка прикрываются!

> но расстояние не может и приходит к нулю, а затем переходит в минусовое значение.


У вас гуманитарчанка. Надежды нет.

> Почему парадоксы могут спокойно существовать нарушая законы логики.


Это какие-такие парадоксы?
223 449168
>>49164
Планковская длина =/= минимальная длина.
Это ограничение нынешних теорий, при которых формулы принимают бессмысленные решения. Если теоряи гравитации доработать, то можно будет уйти на куда меньшие расстояния, чем планковские.
224 449183
>>48550 (OP)
Аноны если вселенной 13 миллиардов лет, как мы можем видить галлактики на растоянии 14 миллиардов световых лет?
225 449185
>>49183
Мы можем видеть галактики, которые сейчас на расстоянии в 30 миллиардов световых лет от нас из-за расширения вселенной.
226 449188
>>49183
Не можем, самый дальний объект это реликтовое излечние, а еще еще за несколько тысачь лет после большего взрыва.
227 449189
>>49185
Есть две точки А и В. Точка А это положение Земли в данный момент времени, точка Б галактика на растоянии 30 миллионов световых лет. То есть ты хочешь сказать, что растояние между двумя этими точками (не Землей и удаленной галактикой) увеличилось до 30 миллиардов световых лет за некий промежуток времени (с момента вылета фатона, до того как он был "увиден")?
228 449190
>>49188
я тоже так думал. Но я смотрел шоу на ютубе об гравитационных волнах( вел ученый участвовавший в проекте) и он сказал о галактике на растоянии 14 миллиардов световых лет. Когда его поправили он сказал об погрешностях и пошутил на тему "верьте всему что я скажу". Шоу на английском было. Я могу дать ссыль но не знаю есть ли смысл.
Я немножко озадачился и решил спросить анонов.
229 449191
Как происходит преобразование тока из 220 в 12, и из переменного в постоянный, можно просто в паре строк сам принцип, амперы от ваттов отличаю.
230 449193
>>49183
Почитай про парадокс "муравей на верёвке".
231 449194
>>49193
Почитал.
Мнэ.
В общем, ответ получен и понят. Двачую всем кто помог приобщиться
232 449196
>>48678
Нет, неправильно понимаешь

>больше


Ты , вероятно, имел ввиду "массивнее"(тяжелее)
Тогда.
Нет, зависимость не такая прямая. Так что если бы Земля была существенно массивнее, позвоночная жизнь могла вообще не возникнуть и, возможно, были бы, например, только водные твари и немного ползающей фигни на суше. Или еще как-то. Но не +50% массы = -50% роста
233 449197
>>49129
Это же очевидно
234 449198
>>49189Скорее всего именно это он и имел ввиду.
Считается, что вселенная расширяется(или расширялась) быстрее скорости света
235 449199
>>49189
Да.
Ну а прям объекты старше 13 с хвостиком миллиардов лет мы видеть, конечно, не можем. И тут нет каких либо прям физических ограничений, просто объектов старше нет. Их нет и рядом с нами, и смотря так далеко мы начинаем упираться в стену, когда вселенная ещё не была прозрачной. Поэтому учённые и пришли к выводу, что возраст вселенной 13.8 миллиардов лет где то. Примерно 400к лет вселенная была непрозрачной и после уже начала становится привычной для нас примерно. Самая старая звезда, которые мы знаем вроде 13.2 миллиарда лет.
236 449200
>>49199
Ну или не возраст вселенной как таковой, а с момента большого взрыва прошло 13.8 миллиардов лет.
Сейчас уж ТБВ не рассматривается просто как было ничто, сингулярность и после пук, и вселенная начала резко расти.
В целом это просто не сходится во многом с наблюдательными данными.

Пока самая популярная теория это инфляционная. Но она сама по себе не делает прям каких либо серьёзных ограничений на размер и возраст до БВ. Может она и бесконечная, а лишь на небольшом участке произошёл коллапс вакуума на более низкое энергитическо состояние, из-за чего и выделилось столько вещества. Может вселенные рождаются постоянные и их число бесконечное, но координатами они никак не связанны.
Что было до БВ и какой размер был и есть сейчас это уже вопрос сложный и спорный. Ибо каких либо ограничений просто нету, что бы можно было сказать, вот если было так, то значит размер конечен, а если так, то бесконечен. И так, и так всё будет работать.
237 449207
>>49135
чето я нихуя непонял, про расширение вселенной я хотябы разговор поддержать могу. Кинь ссылку на видео если есть, я заинтересовался
238 449211
>>49185
Но они уже давно мёртвые. Зачем нам их видеть? Нужно искать те, свет которых мы ещё не видим.
239 449213
>>49211
почему мёртвые то. 13.4 миллиарда лет нашей галактике
240 449218
>>49213
Потому что солнца там уже давно потухли или сколлапсировали, а чёрные дыры давно сжижали вакуум. Свет нашего Солнца дойдёт до других галактик только к тому времени, когда его самого уже не будет.
241 449222
>>49165
Причём тут интернет, если я понять не могу? Информация подаётся как данность, но я не могу её воспринимать просто так.

>Мне неизвестны достоверные научные факты, которые бы утверждали её конечность.


Если есть начальность, значит есть конечность. Расширение вселенной означает, что она не абсолютна.

>Потому что нет конечного номера шага, на котором ты, обкладывая площадь круга квадратиками, преуспеешь в этом.


Но это и есть бесконечность. Почему она может проявляться в исчислениях реальности, но в самой реальности её не существует?

>Это какие-такие парадоксы?


Которые я назвал. Парадокс пространства, когда расстояние полностью сокращается до нуля в определённый момент. Если поставить между стенами стул и сдвигать стены уменьшая одновременно стул, то стены столкнуться несмотря на то, что между ними будет оставаться расстояние стула.
15345282291610.jpg16 Кб, 460x294
242 449223
>>49199

>просто объектов старше нет.


Я конечно, дичайше извиняюсь, но есть ли какие-либо астрофизические теории, обосновывающие невозможность сосуществования звёзд с ЧД на сверхнизких орбитах? Я понимаю в приливные силы, в нестабильность орбит, но если звезда очень плотная, ЧД раскручена сильно-сильно, это же нивелирует данные эффекты? Таким образом, мы можем заиметь хоть 100-триллионолетнюю нейтронную звезду возле ЧД ещё вчера?
15345260647050.jpg370 Кб, 1864x1896
243 449224
>>49222

>Информация подаётся как данность, но я не могу её воспринимать просто так.


Конкретней, пожалуйста.
Если не можешь принять недостаточно обоснованную информацию - это одно. Это потому что пидорасы everywhere, и по людски не объясняют. Но, впрочем, это, к тому же, крайне сложно. Так что тут или суперспец в своей области объясняет, или будешь слушать испорченный телефон теорий.
Если восприятие не выливается в осознание - это другое. Сорт оф расстройство психики, тут уже к психиатру. Излечение (неполное) можешь словить, если задашься какой-то внешне (но не по факту данности) простой задачей, и будешь, денно и нощно к ней возвращаясь, пилить теорию вокруг неё, попутно выясняя, какие соседние научные области она задевает и как именно. Короче, пилишь монографию для себя, полируешь своё осознание. Но это нужно долго и упорно работать, да и не ведаю достоверно, что сработает именно так, как надо.

>Если есть начальность, значит есть конечность.


1. "Начальность" Вселенной - необоснованное допущение. С тем же пылом и задором можно взять за её "начало" нулевой год нашей эры или дату твоего рождения.
2. Из начальности не следует конечность. Точечный заряд имеет то, что ты, по всей видимости, интерпретируешь как начало. Но конец напряжённости создаваемого им поля? Где ты его отыщешь? На "краю" Вселенной? За "краем"? Переход из пространства во время во имя наглядности достигается заменой заряда на фотон, созданный спонтанной эмиссией и направленный куда-нибудь в воид с тем расчётом, чтобы не (никогда) коснуться материи, характеристики -
тензором энергии-импульса, хотя бы. Где твой конец у того, что якобы имеет начало?

> Расширение вселенной означает, что она не абсолютна.


Пока ты не дашь дефиницию своей абсолютности, продолжать беседу суть просто терять время.

>Почему она может проявляться в исчислениях реальности, но в самой реальности её не существует?


Вот факты, свидетельствующие об обратном:
https://en.wikipedia.org/wiki/1_+_2_+_3_+_4_+_⋯

> "The regularization of 1 + 2 + 3 + 4 + ⋯ is also involved in computing the Casimir force for a scalar field in one dimension.[17] An exponential cutoff function suffices to smooth the series, representing the fact that arbitrarily high-energy modes are not blocked by the conducting plates. The spatial symmetry of the problem is responsible for canceling the quadratic term of the expansion. All that is left is the constant term −1/12, and the negative sign of this result reflects the fact that the Casimir force is attractive.[18]"



> Парадокс пространства, когда расстояние полностью сокращается до нуля в определённый момент.


Меня, кстати, по делу щёлкнули в нулевой окрестности моего носа справа и слева здесь >>49168. Едва разрыв не словил.

> Если поставить между стенами стул и сдвигать стены уменьшая одновременно стул, то стены столкнуться несмотря на то, что между ними будет оставаться расстояние стула.


Что за дичь ты тут втираешь? Модифицированный парадокс Ахилесса и черепахи? Как ты осилишь плавно уменьшать стул? А если и осилишь нет, каким это образом топология-то изменится? Как был стул, так и есть.
15345260647050.jpg370 Кб, 1864x1896
243 449224
>>49222

>Информация подаётся как данность, но я не могу её воспринимать просто так.


Конкретней, пожалуйста.
Если не можешь принять недостаточно обоснованную информацию - это одно. Это потому что пидорасы everywhere, и по людски не объясняют. Но, впрочем, это, к тому же, крайне сложно. Так что тут или суперспец в своей области объясняет, или будешь слушать испорченный телефон теорий.
Если восприятие не выливается в осознание - это другое. Сорт оф расстройство психики, тут уже к психиатру. Излечение (неполное) можешь словить, если задашься какой-то внешне (но не по факту данности) простой задачей, и будешь, денно и нощно к ней возвращаясь, пилить теорию вокруг неё, попутно выясняя, какие соседние научные области она задевает и как именно. Короче, пилишь монографию для себя, полируешь своё осознание. Но это нужно долго и упорно работать, да и не ведаю достоверно, что сработает именно так, как надо.

>Если есть начальность, значит есть конечность.


1. "Начальность" Вселенной - необоснованное допущение. С тем же пылом и задором можно взять за её "начало" нулевой год нашей эры или дату твоего рождения.
2. Из начальности не следует конечность. Точечный заряд имеет то, что ты, по всей видимости, интерпретируешь как начало. Но конец напряжённости создаваемого им поля? Где ты его отыщешь? На "краю" Вселенной? За "краем"? Переход из пространства во время во имя наглядности достигается заменой заряда на фотон, созданный спонтанной эмиссией и направленный куда-нибудь в воид с тем расчётом, чтобы не (никогда) коснуться материи, характеристики -
тензором энергии-импульса, хотя бы. Где твой конец у того, что якобы имеет начало?

> Расширение вселенной означает, что она не абсолютна.


Пока ты не дашь дефиницию своей абсолютности, продолжать беседу суть просто терять время.

>Почему она может проявляться в исчислениях реальности, но в самой реальности её не существует?


Вот факты, свидетельствующие об обратном:
https://en.wikipedia.org/wiki/1_+_2_+_3_+_4_+_⋯

> "The regularization of 1 + 2 + 3 + 4 + ⋯ is also involved in computing the Casimir force for a scalar field in one dimension.[17] An exponential cutoff function suffices to smooth the series, representing the fact that arbitrarily high-energy modes are not blocked by the conducting plates. The spatial symmetry of the problem is responsible for canceling the quadratic term of the expansion. All that is left is the constant term −1/12, and the negative sign of this result reflects the fact that the Casimir force is attractive.[18]"



> Парадокс пространства, когда расстояние полностью сокращается до нуля в определённый момент.


Меня, кстати, по делу щёлкнули в нулевой окрестности моего носа справа и слева здесь >>49168. Едва разрыв не словил.

> Если поставить между стенами стул и сдвигать стены уменьшая одновременно стул, то стены столкнуться несмотря на то, что между ними будет оставаться расстояние стула.


Что за дичь ты тут втираешь? Модифицированный парадокс Ахилесса и черепахи? Как ты осилишь плавно уменьшать стул? А если и осилишь нет, каким это образом топология-то изменится? Как был стул, так и есть.
244 449225
>>49223
Гравитационные волны не позволят этому случиться. Из-за них энергия будет слегка каждый оборот уходить, а объекты приблежаться друг к другу. Из-за этого же ЧД или нейтронные звёзды в итоге сталкиваются друг с другом.
15348597259130.jpg38 Кб, 680x558
245 449226
>>49225
Можешь посоветовать литературу по подобному расчёту (для вращающейся ЧД)? Чтобы иметь возможность переконвертировать формулу в максимальную продолжительность жизни мимозвезды.
246 449227
>>49226
не могу
247 449228
>>49226
но ясно что не особо долго относительно жизни вселенной. и вращается тут дыра или нет роли особой не играет
248 449229
>>49224
А что есть достаточно обоснованная информация? Ты сам сказал, что учёные до сих пор гадают. Я зачастую не могу понять именно эту самую обоснованность. Один вопрос порождает другой и так всё упирается в непонимание. Это из-за гуманитарной склонности?

>Начальность" Вселенной - необоснованное допущение.


Что значит необоснованное, если уже вычислено время зарождения Вселенной? Если можно установить время, значит можно установить и пространство. Есть начальная точка.

>каким это образом топология-то изменится? Как был стул, так и есть.


Но он между стенами, которые упираются друг в друга. Можно ли продолжать считать, что стул есть?
>>49168
Как это объясняется? Почему существую такие ограничения и искажения?
249 449230
>>49229

>Как это объясняется? Почему существую такие ограничения и искажения?


Потому что на таких расстояниях гравитация становится уже слишком огромно силой, которая всё в узел сворачивает. Что бы это не было, она должна квантоваться. Как это сделать никому пока не понятно.
250 449231
>>49223
Есть звезды у которых в ядре черная дыра. Вполне реальные обнаруженные объекты и их на удивление много
251 449232
>>49231
Да. Есть ещё Супер-ЧД, одна, может несколько, но здесь я бы пока не стал о чём-то с полной уверенностью говорить...
252 449233
>>49183
В астрономии обычно не измеряют расстояние в млрд. светолет, его измеряют в единицах красного смещения Z.
После чего его переводят в расстояние двумя способами – или это время за которое свет дошёл до нас, или иначе расстояние, которое он до нас преодолел в световых годах (тогда это расстояние не может быть больше 13 млрд. светолет), или же это расстояние до этой галактики прямо сейчас, тогда учитывая ускоренное расширение вселенной, это сиюминутное расстояние может иметь величину до сотни миллиардов светолет, а галактика лететь от нас сейчас со сверхсветовой скоростью (т.е. между нами образуется больше 300 тыс. км нового пространства в секунду сейчас)
253 449234
>>49226
Как я слышал, там расклад примерно такой, что с расстояния на котором время замедляется раза в два, черные дыры сливаются менее чем за секунду.
15380228967560.jpg50 Кб, 333x481
254 449242
>>49234
Охохонюшки.
https://elementy.ru/nauchno-populyarnaya_biblioteka/432656/Chernaya_dyra_v_kinematografe

> Примерно такая же ситуация с сверхмассивной черной дырой Гаргантюа, в систему которой попадают герои фильма. Нолан рассказал Торну, что, по сценарию, Купер отправляется к сверхмассивной черной дыре, там время для него замедляется в сотню раз по отношению к земному, и возвращается он домой, когда дочка уже стала древней старушкой, а он всё так же молод. «Это невозможно, — сказал Торн. — Если время замедляется в сотни раз, то поле тяжести должно быть так велико, что человек будет разорван на атомы, а о том, чтобы вернуться, и речи быть не может. К тому же черная дыра окружена диском плазмы с температурой в миллионы градусов. Купер и его спутники умрут от жесткого рентгеновского и гамма-излучения еще до того, как приблизятся к черной дыре на такое расстояние, чтобы можно было говорить о серьезном замедлении времени».



> «Совсем-совсем невозможно?» — переспросил режиссер. «Не знаю, — честно ответил Торн, — надо составить уравнения, посчитать». — «Посчитайте, — сказал режиссер, — только мне нужно, чтобы в фильме была ОДНА черная дыра, ОДНА кротовая нора и ОДНА нейтронная звезда».



> Торн занялся расчетами, по результатам которых написал потом две научные статьи и упомянутую выше книгу. В книге Торн пишет, что никогда не получил бы этих сугубо научных результатов, если бы не Нолан с его требованиями, потому что ему не пришло бы в голову ради абсолютно неправдоподобного научного предположения исследовать астрофизические явления, создаваемые черной дырой, вращающейся с предельной для нее скоростью. В природе существование таких черных дыр настолько маловероятно, что никто из астрофизиков не стал бы по собственному желанию тратить на подобные расчеты ценное время суперкомпьютеров (да и свое тоже).



> Дело в том, что эффекта, которого хотел достичь Нолан, действительно можно добиться, но лишь в случае, если черная дыра вращается так быстро, что ее горизонт событий, то есть то, что можно считать границей дыры, движется со скоростью, которая меньше скорости света всего на 0,01 миллиметра в секунду! Только тогда пространство-время вокруг нее искривляется и закручивается столь странным образом, что и время замедляется в сто раз, с точки зрения удаленного наблюдателя, и гравитационное поле уравновешивается центробежной силой, благодаря чему и тело астронавта, и корпус космического корабля не испытывают чудовищных перегрузок. В любом другом случае астронавта ждала бы неминуемая гибель.



Просят формулу.
Дают "я так слышу".

> /sci


/thread
15380228967560.jpg50 Кб, 333x481
254 449242
>>49234
Охохонюшки.
https://elementy.ru/nauchno-populyarnaya_biblioteka/432656/Chernaya_dyra_v_kinematografe

> Примерно такая же ситуация с сверхмассивной черной дырой Гаргантюа, в систему которой попадают герои фильма. Нолан рассказал Торну, что, по сценарию, Купер отправляется к сверхмассивной черной дыре, там время для него замедляется в сотню раз по отношению к земному, и возвращается он домой, когда дочка уже стала древней старушкой, а он всё так же молод. «Это невозможно, — сказал Торн. — Если время замедляется в сотни раз, то поле тяжести должно быть так велико, что человек будет разорван на атомы, а о том, чтобы вернуться, и речи быть не может. К тому же черная дыра окружена диском плазмы с температурой в миллионы градусов. Купер и его спутники умрут от жесткого рентгеновского и гамма-излучения еще до того, как приблизятся к черной дыре на такое расстояние, чтобы можно было говорить о серьезном замедлении времени».



> «Совсем-совсем невозможно?» — переспросил режиссер. «Не знаю, — честно ответил Торн, — надо составить уравнения, посчитать». — «Посчитайте, — сказал режиссер, — только мне нужно, чтобы в фильме была ОДНА черная дыра, ОДНА кротовая нора и ОДНА нейтронная звезда».



> Торн занялся расчетами, по результатам которых написал потом две научные статьи и упомянутую выше книгу. В книге Торн пишет, что никогда не получил бы этих сугубо научных результатов, если бы не Нолан с его требованиями, потому что ему не пришло бы в голову ради абсолютно неправдоподобного научного предположения исследовать астрофизические явления, создаваемые черной дырой, вращающейся с предельной для нее скоростью. В природе существование таких черных дыр настолько маловероятно, что никто из астрофизиков не стал бы по собственному желанию тратить на подобные расчеты ценное время суперкомпьютеров (да и свое тоже).



> Дело в том, что эффекта, которого хотел достичь Нолан, действительно можно добиться, но лишь в случае, если черная дыра вращается так быстро, что ее горизонт событий, то есть то, что можно считать границей дыры, движется со скоростью, которая меньше скорости света всего на 0,01 миллиметра в секунду! Только тогда пространство-время вокруг нее искривляется и закручивается столь странным образом, что и время замедляется в сто раз, с точки зрения удаленного наблюдателя, и гравитационное поле уравновешивается центробежной силой, благодаря чему и тело астронавта, и корпус космического корабля не испытывают чудовищных перегрузок. В любом другом случае астронавта ждала бы неминуемая гибель.



Просят формулу.
Дают "я так слышу".

> /sci


/thread
255 449243
В контексте последних событий

> Онищенко призвал бороться в школах с онанизмом


Как современная наука и наукоанон в частности относится к фапу?
256 449244
>>49243
Суперструнщики, тут вам вопрос задали.
257 449245
>>49243
Как к наркотической зависимости.
15381654669520.jpg32 Кб, 195x200
258 449246
Раз уж пошла пьянка с чёрными дырами, вот вам расчёты по устойчивым орбитам ВНУТРИ чёрной дыры по уравнениям Керра-Ньюмена:
https://arxiv.org/abs/1103.6140
https://arxiv.org/pdf/1103.6140.pdf
259 449247
>>49243
Современная наука шлюха капитализма во всех вопросах, что касается человека. Так что тебе в /gay с такими вопросами.
260 449248
>>49242

>просят формулу о которой даже проф. учёные не особо уверенны и должны тратить время


>ожидают какого то другого ответа кроме примерного.

261 449251
>>49245
С таким подходом и к жизни нужно относиться как к чреде наркотических зависимостей, бессмысленных и беспощадных
262 449252
>>49248
Так-то известны вращающиеся дыры, которые делают это весьма резво.
https://www.nature.com/articles/nature11938

> From the relativistic disk reflection, we can determine the blackhole spin parameter. Allowing all other model parameters (such as disk inclination, ionization state and emissivity profile) to vary, we found the minimum spin to be a $ 0.84 at 90% confidence (Fig. 4), where a is the dimensionless spin parameter, a* 5 Jc/GM2 (where J is the black hole’s angular momentum), equivalent to an innermost disk edge at #2.5 gravitational radii. These results are consistent with the ones from previous observations of NGC 1365 (ref. 10) from the X-ray astronomy satellite Suzaku. In that analysis, however, the relativistic model was assumed to be valid, and a complete comparison with the absorption-only scenario was not attempted, owing to the low statistics,especially at high energies (see Supplementary Information for further details).



84% не 99.99999999%, но вопрошатор же и не требует, чтобы компенсация была на уровне человековыживания. Для нейтронных звёзд +-10'000g никаких серьёзных возмущений не вызовут.
263 449256
>>49246
Тоже интересно конечно, но так, это же 2011 год, графики скудные, внешние орбиты не показаны, джеты не показаны, в движении не показывает, но сейчас ведь уже движки тянут и не такое, ну не полностью естественно, а сведя к каким-нибудь вихревым движениям газов по заданным направлениям сводящимися к одному-двум ближе к центру где уже можно особо и не вращать, ну или как вариант мегапрочные маячки внутрь земного ядра как-нибудь забросить и по ним прсмотреть.
264 449257
>>49256

>как вариант мегапрочные маячки внутрь земного ядра как-нибудь забросить и по ним прсмотреть.


Может показать, но не совсем то, смысл тогда если и по простым моделькам можно прикинуть.
265 449264
гайз, не знаю сюда мне или нет, но может поможете. Сам я гуманитарий, дуб дубом, но вот делаю приятелю крафтовый ящик в подарок, хочу на нём пару надписей изобразить, отсюда вопрос что написать? Про него:-гик 1-2 степени. закончил математическую школу, обожает комиксы про научную фантастику( смотрит батл стар галактика, и тому подобное, шелдонов всяких, потом читает хоккинга и главное понимает, также киберпанк всякий, уважает научный атеизм) Может подскажете что изобразить написать на ящике из дерева или нарисовать ? спасибо
266 449266
>>49252
Разница между 84% и 99.99999999 в миллиарды раз больше, а то и больше, чем между 84% и 0.
Представить условия для таких скоростей праткически невозможно во вселенной
267 449267
>>49266
сколько всего больше в миллиарды миллиардов миллиардов раз
15379851121363.jpg173 Кб, 960x752
268 449284
>>49266
Блядь, ну хуле тут упоротые такие.
Я ещё не полностью девятки разместил.
Как влёт оценить отношение дельты скоростей к скорости света, так все горазды.
Как уловить суть и отвечать по существу - я у мамы суперструнщик, примитивные проблемы мне не интересны, отстаньте.

Во-первых, в астрономии вычисления угловых моментов ЧД на поток пока не поставлено, может статься удивительное-рядом, что означенные скорости вполне встречаемы.
Во-вторых, ты в упор не слышишь, что на условия в фильме так-то ПОЕБАТЬ. Поскольку компенсации до уровня g просто не нужны. Рвать нейтронку на части не так просто. Ты бы хоть глянул, что да как, в фильме требование было, чтобы у спустившегося с орбиты на поверхность планеты время шло со скоростью 20 лет за один час. Конечно тут искривления просто невъебенные. Такого банально не нужно.
Единственные оставшиеся вопросы, это найти формулу, которая бы связывала границу эргосферы данной дыровращалки с релятивиским сокращением времени, и для данной орбиты найти скорость потери энергии нейтронной звездой через излучение и гравиволны. И вывести возможность/невозможность триллионолетних монстров. Возможно, можно будет даже под эргосферой этими триллионолетиями оперировать (при специфических параметрах), но я не астрофизик, чтобы что-то тут кудахтать.
В отличие от, очевидно, тебя.
269 449285
>>49284
Я к тому написал, что для того, что раскрутить объект с 99.9998% до 99.9999% энергии нужно потратить в разы больше, чем от 84% до 99.9%.

Поэтому представить условия при которых объект способен так раскрутится просто невозможно.

На фильм мне в целом пофиг. Там и так ясно, что там фигня всё.
270 449294
>>49191
1. 220 в 12 преобразует трансформатор пик1-2, на первичную обмотку идет питание, проходящий переменный ток намагничивания создаёт переменный магнитный поток в магнитопроводе выполненном из ферромагнитного материала, в результате электромагнитной индукции переменный магнитный поток в магнитопроводе создаёт во всех обмотках, в том числе и в первичной, ЭДС индукции, пропорциональную первой производной магнитного потока, при синусоидальном токе сдвинутой на 90° в обратную сторону по отношению к магнитному потоку, выходящее напряжение пропорционально числу витков вторичной обмотки.
2. Переменный в постоянный преобразует выпрямитель представляет собой полупроводниковый мост пропускающий входящий ток в два полу-периода, в каждый из полу-периодов ток проходит два диода, два других заперты, в результате, на выходе получается напряжение, пульсирующее с частотой, пик3вдвое большей частоты питающего напряжения, которое сглаживается до постоянного фильтром на конденсаторе.
15380814594640.jpg72 Кб, 610x384
271 449307
>>49285

>Я к тому написал, что для того, что раскрутить объект с 99.9998% до 99.9999% энергии нужно потратить в разы больше, чем от 84% до 99.9%.


Я не могу не согласиться здесь.

Но, когда речь заходит о ЧД, здесь об "экономичности" энергий говорить не приходится. ЧД в 1365 NGC исчисляется миллионами солнечных масс, при том, что имеет впечатляющий показатель скорости вращения.

>Поэтому представить условия при которых объект способен так раскрутится просто невозможно.


Тяжёлая моя наркомания ниже.
Я не исключаю, что может быть некий механизм типа джетов, который бы разгонял ЧД и дальше. Разумеется, для этого случая условия тоже должны быть сверспецифичные, например, при сверхвысоком заряде ЧД могла бы предпочтительно поедать противоположно заряженные частицы из эргосферы, тогда как оставшиеся, хоть и будучи вовлечены во вращение, по итогу бы отталкивались, передавая дополнительную компоненту вращения из-за вертикального смещения относительно "поверхности", в условиях сильной скрутки пространства имеющую ощутимую горизонтальную составляющую. Пишу и чувствую, что зело наркоманю. Ещё раз, я не астрофизик.


>Там и так ясно, что там фигня всё.


Кроме ебанутого начала и ебанутого конца, там есть занятная середина тоже ебанутая из-за сверхспецифичности подобной ЧД с т.з. Природы, конфигурацию которой высокоразвитая цивилизация, в принципе, могла задать.
272 449312
>>49307

>Я не исключаю, что может быть некий механизм типа джетов, который бы разгонял ЧД и дальше. Разумеется, для этого случая условия тоже должны быть сверспецифичные, например, при сверхвысоком заряде ЧД могла бы предпочтительно поедать противоположно заряженные частицы из эргосферы, тогда как оставшиеся, хоть и будучи вовлечены во вращение, по итогу бы отталкивались, передавая дополнительную компоненту вращения из-за вертикального смещения относительно "поверхности", в условиях сильной скрутки пространства имеющую ощутимую горизонтальную составляющую. Пишу и чувствую, что зело наркоманю. Ещё раз, я не астрофизик.


Думается мне, момент импульса не нарожать так сильно, а он сохраняется.
273 449316
Возможно ли уничтожить вселенную?
Можно ли сделать так, чтобы не появиться никогда в будущем, быть извлечённым из реальности, или мы навечно обречены здесь вариться и ничего не сделать?
9gdCOQkp.png127 Кб, 512x512
274 449318
>>49312

>Думается мне, момент импульса не нарожать так сильно, а он сохраняется.


Моя манягипотеза куда более атакуема контраргументом "а такой-то гигазаряд ты из нихуя генерировать (т.к. его сохранять) собрался?".

>>49316

> Возможно ли уничтожить вселенную?


Я натыкался на препринт статьи, где физики-шизики предлагали регулировать расширение Вселенной манипуляциями с ЧД в масштабах Вселенной. Возможно, тебе сюда. Но искать за тебя лень.

> Можно ли сделать так, чтобы не появиться никогда в будущем, быть извлечённым из реальности, или мы навечно обречены здесь вариться и ничего не сделать?


Тебе в другой раздел. /mg и /re к твоим услугам.
275 449321
>>49318
Если я появился один раз, что мешает появиться мне снова? Среди людей находятся двойники и рождаются близнецы, повторение сознания и ощущений вполне возможны.
276 449323
>>49321
Опиши механизм переноса информации
277 449325
>>49323
Куда?
278 449332
Х хромосома в паре ХХ у женщин взаимозаменяема? Т.е. может ли признак сохраняющийся в Х хромосоме предаться от матери сыну из правого Х в левый Х в паре ХY?
279 449335
>>49332
Не понял тебя. XX хромосомы нельзя выделить на условно левую и правую, они функционально одинаковы, пока некоторые гены не начнут подавлять в какой-либо хромосоме экспрессию генов.
При мейозе у женщин яйцеклетка получает рандомную X хромосому из пары XX.
280 449336
>>49335
Ну вот есть у некой мамки, признак А в одной хромосоме, и признак Б в другой. Будет ли шанс что у сына может появится как признак А, так и Б, или только А? А у внука?
281 449340
>>49336

> Будет ли шанс что у сына может появится как признак А, так и Б


Да
282 449341
>>49336
Если признак приписан аллели определенного гена, то только один вариант.
Если признак признаки приписаны разным аллелям, то (я не уверен) возможно при кроссинговере получение яйцеклетки, в которой X хромосома будут иметь два нужных аллеля гена.
283 449344
>>49341
То есть может быть такое, что признак прописан 2м хромосомам Х и более, и передаётся только от матери к дочери. И если к примеру у единственного обладателя этим признаком родится только сыны, то он может исчезнуть навсегда?
284 449345
А может как-то влиять митохондриальная днк, на что-то помимо самих митохонднрий? И как у митохонрий происходит обмен ген. информации. Они тупо делятся или как? Т.е. если предположить что мутаций не существует, то М-ДНК будет у всех идентична?
285 449346
>>49344
Опять тебя не понял. Все зависит как генами определяется признак, а дальше идет комбинаторика.
У женщин как правило экспрессия генов в одной хромосоме из пары XX подавляется, и признак не может сформироваться из двух разных генов в разных хромосом, но эти гены наследуются.
Дочь наследует от матери только одну хромосому, сын тоже. Сын же может передать материнскую X хромосому своей дочери.
286 449347
>>49345
Митохондия это по-сути отдельный организм в клетке, но поскольку он в глубокой синергии с клеткой, то все ненужные гены митохондрия растеряла. ДНК у нее просто свободно плавает как у безъядерных организмов, и изменения в ней происходят только за счет спонтанных мутаций.
Впрочем митохондии нужны свои мембранные белки и ферменты, которые и закодированы в митохондриальной ДНК. И если есть повреждение генов кодирующие эти белки, митохондии не сможет выполнять свою работу и клетка погибнет. Наследование идет прямое. От материнской клетки к дочерней. Человек же наследует ДНК митохондрии только по матери.
287 449348
>>49346
Ну вот представим что ген есть в одной Х и другой ген в другой Х, оба гена делают признак. У мамки он зародился этот признак в 1 поколении и она родила единственного сына. Ему передался только 1 ген из 2х,. Значит ли это что признак утерян вместе 2 геном?
288 449349
>>49325

> что мешает появиться мне снова


Не знаю, туда, куда ты собрался переноситься. Без этой информации ты будешь не тобой, и никакого переноса как ни крути не получится. И да, не забудь о непрерывности.
289 449351
>>49348
Я уже говорил, что у женщин экспрессия генов в одной из XX хромосоме подавляется. Другими словами одна из XX хромосом блокируются и соответствующие гены в ней отключены. Т.е. призрак не может быть сформирован из разных генов из разных половых хромосом.
290 449352
>>49351
А какой-где подавляется или активируется выбирается комбинаторикой или есть какие-то, где всегда блокируется или активируются.
291 449361
>>49294
Спасибо анон, только ты один детально и по сути стараешься отвечать.
292 449363
Что не так с диалектическим материализмом?
293 449365
>>49363
Противоречивость.
294 449367
>>49363
Все то же, что и с остальными совковыми отрыжками.
1749CD51-7CC1-4916-AA9E-3FA0B07F0313.png7 Кб, 600x600
295 449378
Вопрос жизни и смерти. Не думал, что в жизни когда-нибудь пригодится геометрия. Такой вопрос. Есть треугольник, как узнать длину стороны a, если известна длина стороны b и угол тета?
296 449379
297 449381
Накидайте аргументов в пользу материалистических взглядов на мир. Можно ли в 2к18 считать, что идеализм окончательно развенчан?
298 449385
>>49381
С этим тебе в /ph.

>Можно ли в 2к18 считать, что идеализм окончательно развенчан



У нас тут целая космология, по сути, идеализм чистой воды, облаченный а якобы научную форму. А про общественные науки я вообще молчу.
299 449386
А есть книги по математике для неофитов типа лекций Фейнмана по физике?
300 449389
Можно ли пустырник сделать психоактивным с помощью селекции?
301 449390
>>49389
всё можно сделать с помошью селекции. только вопрос сколько это времени займёт. 10, 10 или 100000 лет
302 449400
>>49386
Целая доска для математики есть. Там лучше и спрашивай.
https://2ch.hk/math/res/29047.html (М)
303 449415
Как устроен компьютер? Что такое RISC и CISC
,архитектура фон Неймана,сумматоры и мультиплексоры? Если что ,может порекомендуете какую литератур читать по данной теме?
304 449416
>>49415
это не /sci
305 449418
>>49416

>это не /sci


Ммм,тогда на какую доску с такими вопросами можно идти?
306 449419
>>49418
на хуй палю годноту быдлу
307 449420
>>49418
есть доска про железо
308 449421
>>49419

>Дэвид М. Харрис Сара Л. Харрис Цифровая схемотехника и архитектура компьютера


Уебанская конфигурация мешает нормально читать.А так про это книгу знаю,и даже пробовал читать ,но со смартфона с этой хренью читать просто вырвиглазно.

>Сворень Р.А. Электроника Шаг За шагом. Практическая энциклопедия юного радиолюбителя


Ну ты конечно загнул.Эта книга давно прочитана и даже лежит в бумажном варианте на полке.Мне конкретно в архитектуру компа,работу процессора хочется углубится,но имхо,толковых книг для себя не нашёл.Ещё предложения буду,хуйло?
309 449422
>>49420
Кинь ссыль.
310 449425
>>49421

>>Ну ты конечно загнул.


бля, я ясновидящий чтоли

>>со смартфона


нихуя не удобно делать, нужна норм читалка, лёгкая и качественная
311 449426
>>49421

>>Ну ты конечно загнул.


бля, я ясновидящий чтоли

>>со смартфона


нихуя не удобно делать, нужна норм читалка, лёгкая и качественная
312 449438
>>49425
>>49426

>со смартфона


>нихуя не удобно делать, нужна норм читалка, лёгкая и качественная


Ну и где мне её взять? Ладно,хрен с ней, буду пробовать сменить ориентацию страницы другими способами. В любом случае, спасибо за литературу.
313 449463
>>48628

>Либо тело в принципе не создано для дальнейшего развития, как у дельфинов



Вот сейчас обидно стало. мимо-дельфин

Объясни, почему не создано для развития? Откуда инфа, какие критерии?
314 449478
>>49463
Что бы дальше развиваться мозгу ты должен уже куда активнее что-либо конструировать сам. Одно дело заманить рыбку в ловушку, с этим дельфины справятся, но как они построят капкан, в воде то? Не только нет причин для роста мозга, ибо дельфины в своей нище уже отлично справляются и рост интеллекта это лишь лишняя трата энергии, так и возможностей этим ростом воспользоваться не будет.

Там могут постоянно появлятся более умные дельфины, но ничем особо лучше в своём деле они не показывают, как и более глупые не показывают себя сильно хуже, поэтому размножаются одинаково и всё это смазывается.

Или у динозавров, что бегали вертикально. У низ мозг расти не мог, ибо голова начала бы сильно перевесить, а в вертикальную форму они перейти не могли. Ибо перед этим тоже куча неэффективных стадий перехода.

И если так всё и сложить, то приматы, что хватаются за ветки и были идеальным фундаментом для создания интеллекта.
Ибо им легко было стать прямоходящими, а значит и растить головной мозг. У них был самый лучшие конечности для манипуляций, что позволяло им этот интеллект дальше в крафте использовать.

Всё что нужно было этой обезьянке так это выйти из безопасных для неё джунглей в степи и саванны. Где уже и выживали лишь умные. В самих джунглях им тоже резона умнеть нет же.
315 449480
>>49478

>Динозавры бегали горизонтально.


фикс
316 449481
>>49478
Понимаю твои аргументы, но как из них следует, что "тело в принципе не создано для дальнейшего развития"?
А если условия среды изменятся (что уже сейчас происходит вследствие человеческой деятельности)?

Да, и раз уж о дельфинах речь зашла, у меня еще такой вопрос к наукоанонам: почему и для чего они выкидываются на берег?
Ведь для самоубийства им достаточно заплыть чуть глубже и перестать дышать (сознательное дыхание ведь)?
317 449483
Есть один охлаждающий элемент.
Анон, поясни: какой его вариант эффективнее?
318 449484
>>49481
Хах, ну в этом плане да, дельфины могут выйти на сушу, отрастить ноги-руки и дальше развиваться.

Но я в том плане писал, что на этапе предразумной жизни им мозг развить дальше в разы проще, чем отрастить руки, например. Но мозг расти не будет, ибо рук нет.

Выбрасываться на берег просто проще. На дне нужно прилагать усилия. Попробуй сам себя утопи. А так просто перестаёшь плыть и даёшь течению уносить тебя куда угодно.
noamchomsky.jpg335 Кб, 960x1200
319 449488
Поясните за пикрил, великий мыслитель и гуманист, или поехавший маразматик?
320 449490
>>49488
Это как с Полингом, научные труды охуеные, но стоило ему заквашриться об политоту, то все приехали.
321 449494
Для чего нужна температуропроводность? Теплопроводность и теплоемкость понятно для чего. Может есть какие формулы и вычисления, где температуропроводность используется?
322 449497
>>49494
Вообще, все вопросы которые ты задал, полностью покрывает статья на вики.
323 449499
>>49497
Я там не понял нихера. Меня единица измерения смущает: если у остальных это тепловая мощность или количество тепла в размер на температуру, то тут просто площадь в секунду.
Допустим, есть два тела с одинаковой теплопроводностью, но с разной температуропроводностью α2 = 10α. На что это может повлиять? Ну и допустим, что эти тела передают тепло от одного тела другому.
324 449500
>>49490
Швятую омерику посмел критиковать?
325 449501
>>49500
срывать покровы, которые умным и так очевидны, а тупые всё равно не поймут.
так же попытка рационализировать общественную жизнь, фрикавство.
326 449511
>>49501
Очевидны, но противоречат официальным идеологическим установкам. А значит не настолько очевидны для рядового американского пидорахена.
327 449512
>>49511
Не очевидно, потому что общество мыслит в категориях выгоды, а не правды. Или точнее куда скорее поверит в то, что им приятнее, выгоднее и так далее. А иначе будут уходить в жуткое отрицалово, хоть там о геноциде очевидным будет речь идти.

И не нужно думать, что на Западе какие то другие люди, что любят просто жутко каяться. Если появится новость, где мафия педофилов-гомосексуалистов по всему миру ест детей с очевидными доказательствами, то оно так же уйдёт в жуткое отрицалово по начало. Просто праваки детьми не занимались и их воспитанием, вот и воспитали их другие люди, но натуру человеческую никаким воспитанием не изменишь.

Ну и самая главное, такие критики мессианской американской политики не столько выступают против неё, сколько недовольны, что этим мессианством занимается в недостаточной мере и в первую очередь думают о шкурном интересе, а не о ценностях, свободах и так далее. Поэтому и в них пидорахенская суть так же сидит, как бы они рационализацией не давили.
328 449517
>>49512
Не обижайся, но рассуждения уровня подростка, который как в историю, так и социологию даже и вникать не пробовал.
329 449533
>>49517
Так это не о истории и социологии пост. А о том, что такие люди те же пидорахены, просто нужно найти правильные кнопки. Это в США они чувствуют себя в безопасности и могут спокойно рассуждать обо всём и не парится. Но если начнут чувствовать угрозу своей безопасности, то сами же активно и начнут призывать к войнам, дабы статус кво восстановить.
330 449536
>>49533
Не он, но - ты наивен не в хорошем смысле, как будто ты вообще книг не читал и все рассуждения построены на твоём пребывании в echo chambers вроде двощей.
331 449539
Ребятки, у меня очень тупой, но очень прикладной вопрос, который связан, наверное, больше с практикой, нежели теоретической физикой.

Область - ультразвуковая обработка.

1. Мощный ультразвук справится с физическим разрушением пластика? Достаточно толстого (5мм), но хрупкого на излом. Или только поплавит, сколько мощности не добавляй?
2. Оснастка - я обратил внимание, что всё, кроме ванн ультразвуковых, использует очень точечный инструмент, а мне нужно сразу пластину 20 см шириной, длина похуй, ну там 1, 5 10 см. Так вот - поддоны в ультразвуковых ваннах на сколько мощные? На сколько применима вообще "пластина", а не "наконечник" для разрушающих воздействий? Может стоит подумать об индивидуальном заказе пластины не гладкой с рельефом?
4-2-cautions-about-correlation-and-regression141.png89 Кб, 800x656
332 449563
Чем индукция отличается от экстраполяции?
333 449582
>>49563
Тут не различия, тут экстраполяция использует индукцию в своём построении.

При том сама по себе индукция может быть часто ошибочной и нужно быть аккуратным с ней, а экстраполяция и подавно.
Но весьма любима британскими учёными, ибо с ней можно вывести любой удобный факт и натянуть легко сову на глобус под грант.
334 449603
Есть ли у гравитации "скорость распространения"? Она быстрее скорости света?
335 449604
>>49603
Равна скорости света вплоть до 15-го вроде знака после запятой, с большей точностью пока не измерили.
Pole1.gif3,3 Мб, 352x200
336 449606
>>49604

>равна скорости света


Глупые людишки.
337 449610
Что будет, если на сверхсветовой скорости включить фары?
338 449611
>>49610
что будет если ты съешь сам себя?
339 449618
>>49611
>>49610
Это коанов тред?
340 449619
>>49611
В другой в вселенной, с другими законами позволяющими разогнаться быстрее света?
341 449620
>>49619
В таком случае и свету ничего не помешает разогнаться быстрее скорости света. Поэтому скорость света твоих фонарей будет твоя сверхсветовая скорость+базовая скорость света это вселенной
342 449621
>>49478

Про динозавров дурной аргумент. Ничто не помешало потомку динозавров - фороракасу иметь огромную и очень тяжелую голову, она конечно была тяжелой не из за мозга, а из за толстенного черепа и огромного клюва. И это прит ом, что у них не было хвоста в качестве противовеса. Далее, ничто не мешало некоторым видам дромеозаврид жить на деревьях, в связи с чем их структура скелета могла измениться. Не забывай, что палеонтология выявляет меньше пяти процентов видового разнообразия на момент геологического периода. Так что я бы не стал исключать возможность эволюции интеллекта у архазавров, но нужно учитывать .что продукты интеллекта, и особенно цивилизации намного хуже сохраняются на протяжении геологических отрезков времени.
343 449622
>>49621
Там пример был именно про рапторов, которые в теории показывали какие то зачатки разума. Не приматы, но смекалка какая то была.
344 449623
>>49622
Чем их разум был не разум, а разум приматов-разум?
345 449625
>>48646

А кто им помешает вырастить лёгкие-то? Более того, вот кокосовый вор замечательно себя чувствует в крупноразмерном классе, именно с такой эволюционной обновкой. Проблема членистоногих не в этом, а в экзоскелете и пневматической системе движения конечностей. Экзскелет тяжелый, конечности-трубки малоэффективны с увеличением размера. Ну и конечно не подходят как манипулятор для мелкой маторики. А без неё не возможен разум человеческого типа.
346 449627
>>49622

Я понимаю, и об этом и говорю. Форма тела им не помешала бы ни перейти к древесному способу жизни (а насколько я знаю, даже из открытых видов дромеозаврид, некоторые подозреваются в таковом), ни к увеличению головного мозга, тяжесть головы скомпенсируется хвостом и усилением плечевого пояса и мышц шеи. Более того, я читал, что пальцевой опор при бипидальности, намного лучше для локомации чем стоповой, соответственно часть ресурса мозга была бы освобождена от сложной системы, позволяющей нам держать равновесие в разных положениях. Передние лапы свободны, плечевой пояс вполне может раздаться, все условия для эволюции разума, кроме очень агрессивной хищной мегафауны, пожалуй. Отогнать дубинами и острыми палками условного тигра можно, огнём тем более. Отогнать тиранозаврида не получится ничем. А крупные хищники были распространены повсеместно.
347 449628
>>49627
Слишком далек этот поход и слишком сильны изменения при этом, что мы больше и не сможем их динозаврами называть.

В теории то, они, конечно за сотни миллионов лет могли бы измениться сильно и в итоге разумная жизнь могла бы появится на их базисе.

Но там речь была о том, что вот у вида мозг стал развиваться и расти, а потом причины по которым дальше он уже не может расти. Что бы дальше он развивался нужны кардинальные изменения, что по сути уже новый вид совсем.
348 449629
>>49628

>Слишком далек этот поход и слишком сильны изменения при этом, что мы больше и не сможем их динозаврами называть.



Чего-чего? Динозавры - это надотряд. В него входят настолько разные создания по форме и виду, что надо очень постараться, чтобы получившееся существо нельзя было назвать динозавром.

>Но там речь была о том, что вот у вида мозг стал развиваться и расти, а потом причины по которым дальше он уже не может расти.



Да ,я умею читать. Но я привожу тебе пример архозавра, с очень тяжелой головой, и никаких кардинальных изменений для того, что её носить ему не потребовалось. Причём в отличии от страуса например, фараракос имел и весьма развитые передние конечности, а значит сильный плечевой пояс. Всё что для этого нужно, более мощная шея и мышцы спины. Всё.

Пойми, пример про дельфинов совершенно верен, но в водной среде, невозможно возникновение разума человеческого типа, нет задач. Кроме того, уже некоторое время в научной среде доминирует версия, что интеллект дельфинов очень сильно переоценён, огромный и сильно развитый мозг оказалось отдаёт гигантскую долю своей активности на расшифровку данных эхолокации, и ещё какую-то срань (я особенно подробно не разбирался, если честно), в итоге интеллект дельфина примерно на уровне собаки, но никак не примата.
А с дромеозавридами это нифига не так. Нет никакой принципиальной причины (что касается строения тела, конечно), которая помешала бы им развивать мозг. По крайней мере по аналогии с приматом.
349 449631
>>49629
Проблема ещё в эффективности и целесообразности. Эволюция не имеет цели, что понятно. Разумная жизнь не является обязательным итогом для неё и никакой вид за этим гнаться без профитов не будет.

Эволюции просто менять формы и размеры, но переходы из одного, допустим лап с когтями, которыми цепляются за кору и к хватательным в разы сложнее. Просто потому, что нет нормальной переходящей стадии. Если у енота появится мутация, которая сделает его лапу более похожей на человеческую, то он станет не эффективен во всём. Не сможет и цепляться уже нормально когтями, но так же и не сможет и хватать ею ничего нормально. Ни то, ни сё, поэтому такого перехода ожидать в разы сложнее, чем просто рост мозга. Это для эволюции так то в разы проще, ибо постепенно и никаких кардинальных изменений.

С ростом мозга же, рост головы куда более значительный, чем просто с ростом какой-нибудь шишки на лбу или рога.

Растёт абсолютно ибо и при этом и мозг надо так же защищать активно, поэтому не можешь себе позволить тонкий и легкий череп. Его размеры и толщина так же должны вырасти значительно. Все эти пазухи у нас же и появились, что бы вес черепа снизить.

И опять. Речь не о принципиальных причинах, речь о принципиальных ограничений исходя из теории эволюции, в которой выживают лишь приспособленные. Ибо все подобные изменения приспособленность лишь снижают.
И приматы и были тем исключением, где рост интеллекта приспосабливаемость лишь увеличил.

Ну а если наука дальше разовьётся, то в теории можно и разумных кошаков вывести. Серьёхных ограничений тут, конечно, нет. Но в дикой природе такое невозможно.
350 449632
>>49631

>И приматы и были тем исключением, где рост интеллекта приспосабливаемость лишь увеличи


Ну сейчас интеллект наоборот стал снижать приспособленность. (У человека)
351 449633
>>49632
Где? наоборот дал расплодиться просто хрен знает за последние сто лет. Может быть приспособление в дикой природе, но это в целом не важно.
352 449634
>>49632

Мы уже не обитаем только в биосфере. И всё большее повышение интеллекта способствует выживанию и успеху в антропосфере. Вид здравствует и процветает, так что наша стратегия очевидно удачна.

>>49631

>С ростом мозга же, рост головы куда более значительный, чем просто с ростом какой-нибудь шишки на лбу или рога.



Во первых, можно пруф? А во вторых С ростом рога, тебе нужно не только сам рог вырастить, но и крепления под него, кости обеспечивающие сохранность мозга при столкновении (рог ведь нужно ещё и использовать), усиление защиты глаз и рта, и далее в таком роде, а это ведь тоже дополнительная масса. У тех же самых фороракосов голова весила чуть ли не четверть тела, их основным способом выживания был клюв ,которым они забивали добычу. Почему ты думаешь что эволюция не могла пойти тем же путём, относительно мозга?

>И опять. Речь не о принципиальных причинах, речь о принципиальных ограничений исходя из теории эволюции, в которой выживают лишь приспособленные.



Ты излишне упрощаешь. Значение имеет всё, удача, окружающая среда, банальная случайность.

> И приматы и были тем исключением, где рост интеллекта приспосабливаемость лишь увеличил.



Исключение - совершенно непримиримое в эволюции слово. Если что-то произошло один раз, на достаточно большом промежутке времени это повторится с вероятностью в единицу. Тем более, нынче на земле несколько успешных видов, которым помогает именно высокий интеллект, помимо человека. Значит это вполне себе универсальный инструмент. А при наличии подходящих условий, инструменты у животных улучшаются, до того момента, пока вид не становится сверх специализированным, живущим только за счёт этого инструмента, в условиях определённой внешней среды - это почти всегда наиболее результативная стратегия.
353 449639
Вопрос такой: правильно ли я понимаю, что легкие от жабр принципиально не отличаются, в том плане, что и то и другое - сборка мембран большой площади, через которые гуляют из-за разности концентрации кислород и углекислота? И что человек задыхается с заполненными водой легкими по причине того, что в воде в 30 раз меньше массы кислорода на литр, и (ну, условно) если бы у него площадь внутренней поверхности альвеол была до 3600 квадратов, то всё бы было ок? И что воздуходышашие как-то дохуя кислорода жрут в сравнении с рыбами.
Александр Марков Открытия в эволюции. Итоги 2017 года (1).mp413,8 Мб, mp4,
640x360, 5:25
354 449640
>>49633
>>49634
В человеческой популяции очень сильный отбор отрицательный по интеллекту. Настолько сильный, что в самом геноме появились метки "это делает отбор".
355 449661
>>49640
Так в том и суть. Эволюции плевать на разум и интеллект, если возможно, она от него тут же избавляется и вид деградирует на более примитивный, что не раз было в истории. Ну не в человеческой, а в целом. Куча околоразумных приматов, что не развились, а наоборот, деградаровали.

Люди конкурировали активно между собой всю историю, где ума требовалось всё больше и больше. А если построят райский мир и никак не будут следить за отбором в нём, то вид не сможет не деградировать.
356 449662
>>49661

> не будут следить за отбором в нём


Фашист? Фашист.
357 449663
>>49661
Потому что нехуй дрочить на интеллект, бородавчатый ты унтерок. Нет в интеллекте нихуя хорошего.
Для людей важно что? Социум, кооперация, мы общественный вид, заваливающий проблемы толпой, точка. Интеллект отрывает тебя от толпы, ты начинаешь преследовать индивидуалистические цели и всё, виду ты не всрался, досвидос.
Доходчиво?
358 449664
>>49663
Ты так говоришь будто тупые преследуют общественные цели.
359 449665
>>49664
Разумеется. У них недостаточно индивидуальности для преследования личных целей, они делают то же, что все.
360 449666
>>49665
Чушь.
Тупые и в социальку и общество хреново могут.

Нигилизм же, о котором ты явно толкуешь это не следствие ума, а его болезнь.
361 449667
>>49662
У фашистов ничего и не вышло бы.
Но я реалист. И это, что не избежать. Хорошая жизнь мозг разжижает. Но и слишком стрессовая на всю жизнь ему на пользу не идёт.

Да и про отбор я писал, никак про в печь всех тупых.

Хотя если ты будешь поддерживать развитый интеллект лишь ради интеллекта, то это понятно дело никуда не приведёт. Должны быть задачи, а иначе и тупой поймёт, что такая селекция на райском острове бессмысленна и жестока. И продолжать её смысла нет.
362 449668
>>49582
А можно подробнее? Ну типа "экcтраполяция - это то и то, а индукция - это и это. Отличается так и так"
363 449669
>>49665
Да и такие явления, как жадность или альтруизм не столько с интеллектом связанны, сколько с гормонами.

Когда человек перестаёт чувствовать свою связь с группой, то тогда уже и меньше склонен ей помогать. Интеллект или разница в ценностях, профессиях, интересах и могут повлиять на разрыв с этой группой. Но винить в этом прямо ум глупо. Ты можешь разорвать отношения с группой будучи олигафреном, из-за того, что ты болеешь за спартах, а твой братуха любит хоккей.
364 449670
>>49667
Отбор предполагает ограничивание рождаемости. А это фашизм
365 449671
>>49670
Или патриархат. Ведь папка не выдаст свою дочь за олигафрена.
366 449673
>>49666

>Тупые и в социальку и общество хреново могут.


Я не говорю о тупых. Я говорю об iq 100, которые несомненно могут в социалку, потому что являются её основной составляющей. А тупой - это то же самое, что и умный.
367 449674
>>49670

>Отбор предполагает ограничивание рождаемости. А это фашизм


А естественный отбор? Естественный фашизм?
Выходит фашизм это естественно.
368 449678
>>49665

>они делают то же, что все.


стадное чувство никто не отменял
369 449687
>>49673
просто гениальность часто сопровождается различными расстройствами, ну и в целом им сложно находить себя в обществе.
Но так же есть и гении семьянины, так что глупо пинать на это.
"Я такой умный, поэтому и живу с мамой и ни с кем не общаюсь больше". Так это звучит.
370 449704
Почему у всех литий-ионных аккумуляторов(из одной ячейки состоящих) номинальное напряжение 3.7в? Это вызвано химией процесса? И почему ей это вызвно.

Другое, как я понимаю, напряжение в принципе сделать нельзя?
371 449705
>>49704

>Это вызвано химией процесса? И почему ей это вызвно.


Да, именно так.
Почему? Какой из почему твой?
"Почему хим. реакции вызывают ток?
Почему потенциал у этой реакции именно такой?"
Хочешь напряжение больше, подключай последовательно.
373 449736
>>49734

>o-religii.ru


Давай лучше сам. Слушать софистов попов не желаю.
изображение.png5 Кб, 825x86
374 449737
Подскажите сайт со статьями по медицине. Нужны последние данные по тенденции к ожирению, можно на рус., англ. дата публикации не более 5 лет (наверно).
375 449744
Что будет, если сделать белок из неиспользуемых природой аминокислот? Будет ли он биологически активен, усвояем людьми, ядовит?
376 449746
>>49736
Вкрации, у пауков есть алгоритм построения паутины закодированный в генах. Разве это могло образоваться само?
>>49737
https://scholar.google.ru/scholar?as_sdt=1,5&q=overweight&hl=ru&as_ylo=2014
377 449747
>>49746

>Разве это могло образоваться само?


Что тут сложного то. Они делают примитивую сеть.
378 449748
>>49746

>у пауков есть алгоритм построения паутины закодированный в генах. Разве это могло образоваться само?


Пауки у которых паутина была хуевой, меньше жрали и просто вымерли.
Ты можешь зарандомить на компе паутину, сделать аналог генов, которые кодируют форму паутины, а после начать игру на выживание у твоих электронных пауков, не забыв рандомно стрелять мухами и включив процент мутаций. В конце концов у тебя на экране будет паутинка прям как у паука.
Чайникэмалированныйнагазовойплите.JPG144 Кб, 1284x1600
Посоны, в детстве термодинамику проебал. 379 449758
Вопрос следующий. Сча попробую сформулировать.
Короч есть литр кипяточка. Есть два варианта:
1. Подождать 15 минут и разбодяжить до 10 литров водой комнатной температуры.
2. Разбодяжить сразу и попдождать 15 минут.
В каком случае вода будет теплее?
380 449759
>>49748
В паутине только частота соединений влияет, а те которые плели не очень могли что-то другое развивать, есть и летающие пауки, и на плёночки воды которые держатся.
381 449761
>>49758
Нужно формулы из школы вспоминать. Но это ты можешь и сам
382 449762
>>49758
Сразу будет теплее, так что лучше подождать, наверное.
383 449763
>>49758
Хотя плохо прочел. Второе конечно.
384 449764
>>49761
Какие формулы? Я же говорю, проебал термодинамику. Вообще не ебу, что там за формулы и как они гуглятся.
>>49763
Если так подумоть, вода отсывает не линейно, а экспоненциально. То есть, в чай надо лед кидать не сразу, а подождать, пока кипяточик сам немного остынет.
385 449765
>>49764
Это классика, это знать надо.
386 449766
>>49764
да это одна из первых формул, которые школьники вообще изучают вроде бы
387 449767
>>48550 (OP)
Учеными предложено много гипотез, касающихся исчезновения динозавров 65 млн. лет назад. Многие считают наиболее вероятной причиной этой катастрофы последствия падения гигантского метеорита на полуостров Юкотан (Мексика).Скорее всего, считают исследователи, падение метеорита вызвало сильнейшие землетрясения и цунами, а поднятая пыль закрыла Землю от солнечных лучей, вызвав гибель растительного и животного мира.
Рассматривается еще много причин происшедших катаклизмов.
Мне кажется можно объяснить гибель динозавров, Всемирный потоп, исчезновение Атлантиды гораздо проще.
Представим себе, что 65 млн. лет назад Земля вращалась с много большей скоростью, чем сейчас. Этим можно объяснить огромные размеры динозавров, людей (атлантов), а также растительного мира.Также представим себе, что в Землю врезался огромный метеорит, причем врезался против направления вращения Земли. Масса метеорита была настолько велика, что вызвала значительное замедление скорости на плечи водрузиливращения Земли.
Что произошло в этом случае?
1. Резкое увеличение веса представителей животного мира, приведшее к их быстрому вымиранию. Их погубила гравитация. Представьте, что вам на плечи водрузили груз порядка 100 кг - долго ли вы выдержите?
2. Океанические воды несколько раз обогнули земной шар (Всемирный потоп).
3. После успокоения водного пространства стали затоплены значительные территории суши (изменилась кривизна поверхности океана вследствие гравитационных воздействий). Так была затоплена Атлантида.
По моему эта гипотеза наиболее просто и реально освещает причину гибели динозавров и исчезновение Атлантиды.
388 449768
>>49767
Поэтому Эйнштейн и предлагал, чтобы вместе с нацеливанием ядерного оружия по крупным городам, была система и от таких тем тоже.
389 449769
>>49765

>Я сам не знаю


Ну так бы и сказал, хули ты.
390 449772
>>49767
Нет. Но идея интересная.
391 449777
>>49767

>гибель динозавров, Всемирный потоп, исчезновение Атлантиды


Одновременно?
392 449780
>>49705

>Почему? Какой из почему твой? Почему хим. реакции вызывают ток? Почему потенциал у этой реакции именно такой?


>Почему у всех литий-ионных аккумуляторов номинальное напряжение 3.7в? почему ей это вызвно.


@

>Хочешь напряжение больше, подключай последовательно.


>из одной ячейки состоящих

393 449782
Если все частицы есть возмущения одноименного поля, то почему:
1)поле может иметь возмущение кратное этой частице и состоящее из их целых частей, а не иметь одно большее или меньшее возмущение?
2)почему эти квантовые возмущения такие устойчивые, а не рассеиваются/расплываются в этом/по этому полю? Мне это кажется чем-то вроде "потому что потому что". "Она квантовое возмущение поля потому что она устойчива, она устойчива, потому что она квантовое возмущение поля."
394 449783
Аноны, скиньте ссылку на запись трансляции "Учёные против мифов", не хочу ждать полгода пока они выложат, я знаю она на Ютубе сохраняется.
395 449797
>>49782
Ты не правильно понимаешь сути частиц. Частицы не возмущение поля, это квант поля. И более того, частицы это отражение квантового поля на некоторой сложной (классической) системе.
Вообще это все обсасывается в курсе КТП с сосуществующим мат методам. Сюда не впихнешь в ответ пол курса, а вкратце нельзя.
image.png73 Кб, 883x128
396 449801
Не нашёл тред научных шюток, так что пусть полежит здесь.
Н. Лейн. Вопрос жизни. Энергия, эволюция и происхождение сложности
гагарин-плоская-Земля-потомки-4202482.jpeg46 Кб, 480x689
397 449815
>>49758

>В каком случае вода будет теплее?


Всё зависит от температуры и давления в твоей комнате.

Технически, тебе нужно оценить следующее:
E1/V1 сравнить с E2/V2
E (исходная) (энергия)=p (плотность)V(объём)tt (теплоёмкость)T (температура)
1 кг/литр
1 литр 4200 дж/(кгградус)100 градусов

ET (энергопотери тепловые) = S (площадь контактной площадки сосуда)
tk (средний коэффициент теплопроводности среды; если сосуд не подвешен, то у тебя верней всего две среды, одна из них - на коей сосуд зиждется) dT (разница температур и теребишь её логарифмический интеграл) t (время)

Скорее всего, у тебя всё прекрасно с термодинамикой, но ты троллируешь sci/, прекрасно понимая, что без конкретики по площади сосудов и степени из закрытости (парообразование же) по факту хуй ответишь. Это как самолёт на движущейся платформе. Молодец, чо.

Лучше ответь мне сам за физику такого процесса:
При отключении газовой конфорки с кипящей на ней кастрюлей воды внезапно происходит куда более интенсивный (кратковременный) выброс пара из воды. За счёт чего это происходит?
399 449820
>>49816
К чему это?
400 449827
Как плазма в токамаке не плавит его? Там же архинеебическая мощность теплового излучения должна быть. И вообще сколько граммов вещества единовременно варится, например, в итере будет?
401 449828
>>49827
Если бы не плавило, то были бы уже работающие токамаки, но их нет. В теории магнитное поле не должно давать ей касаться поверхностей реактора.
402 449829
Я тут подумал что всё искусство наебалово мозга и его не существует, вся худ. литература семантика для попытки объеба и управления людьми другими людьми.
Все картины это культура и ничего более.
Музыка вся просто совпадает в головах, если бы не, то не было бы концертов.
Тоесть интелектуальная мастурбация и всё.
И надо просто исходить что привык тои ешь и нет элитности и всё. Как и ума от гуманитарных наук кроме небать доверчивого
403 449830
>>49828
Так я не про не соприкосновение плазмы со стенками, а излучение от него. Всякое нагретое тело излучает, чем выше, тем сильнее. Чем это нейтрализуется?
404 449834
>>49820
Здесь не любят читать исходные вопросы/запросы. Зато любят отвечать (не в тему и довольно часто неправильно).
К этому.
Просто нагорело.
405 449836
>>49834
Я в тему ответил. Просто он не понял, что я имел ввиду, что больше это напряжение кроме как подключением еще ячеек не сделать.
А отвечать на пространный вопрос "почему солнце встает на востоке". Без уточняющих вопросов это гиблое дело. В конце концов выяснится, что человек считает землю плоской, и вообще я какую-то хуйню ему написал.
406 449841
>>49836
Ты не вчитался в вопрос, в нём всё есть.
407 449843
>>49841

>Это вызвано химией процесса?


Да.

>И почему ей это вызвно


Отсылаю к книгам по термодинамике. Там всё доступно. Сюда писать это слишком объемно.
408 449846
>>49836
Если ты >>49780-кун, то я же не критиковал, а просто излагал свою интерпретацию, почему оно так происходит. В любом случае, проехали.
image.png8 Кб, 619x412
409 449864
Почему энергии, запасённой в растянутой пружине будет, будет меньше, чем понадобится, чтобы поднять большой камень с помощью длинного рычага?
410 449866
>>49864
Не будет меньше. Закон сохранения энергии нерушим.
У пружины мощности не хватает, а не энергии.
411 449885
>>49767

>1. Резкое увеличение веса представителей животного мира, приведшее к их быстрому вымиранию. Их погубила гравитация. Представьте, что вам на плечи водрузили груз порядка 100 кг - долго ли вы выдержите?



Ну давай теперь подумаем. Как выжили млекопитающее, другие архозавры (птицы, крокодилы), черепахи и так далее. Но в то же время, от чего же вымерли тогда морские ящеры? Короче хуйня какая-то. Я уж не говорю о том, что ты зачем-то мешаешь в кучу динозавров и высокоразвитых гомонид которые вряд ли могли существовать в одно время. И самое последнее, насколько я знаю (тут могу ошибаться), вращение оказывает весьма небольшое влияние на силу притяжения. А, ещё, почему-то кайнозойской мегафауне быть весьма и весьма крупной, пусть и не размеров самых крупных динозавров.
412 449886
>>49885

>А, ещё, почему-то кайнозойской мегафауне быть весьма и весьма крупной, пусть и не размеров самых крупных динозавров, это не мешало.



быстрофикс
413 449887
Геодезисты, помогите вывести формулу для перевода точек в географических координатах широта-долгота в точки на плоскости азимутальной проекции в полярных координатах радиус-угол. Имею скрипт на питоне, который из файла берёт пары координат и бацает их в другой файл уже в полярной системе. Делаю явно что-то не то, так: выходит растянутая залупа. Пробовал стереографическую и ортографическую проекции, херня оба раза. Как делать, двач?
image.png1,5 Мб, 1280x960
414 449888
>>49887
Вот "рассчёты" для ортографической, она на первом пике в посте >>49887
415 449893
>>49887
А чо у тя ноль в гринвиче?
416 449894
>>49885
динозавры не вымерли, а эволюционировали в птиц. а сами по себе тупиковой ветвю были. слишком большие и зачем такое тело кормить.
417 449897
>>49887

>Как делать, двач?


Долгота это угол; инвертированная широта, множенная на коэффициент это радиус - получается азимутальная полярная проекция.
418 449900
>>49885
Метеорит сейчас вроде бы не самая популярная теория. Ибо вымирали они всё же в течении долгого времени, а не катастрофа разовая это была. На то время так же и цветочные растения активно плодиться начали, которые динозавры есть не могли.
Так что вымерли они скорее всего из-за комплекса проблем и в целом тупиковым видом были. Кто из тупика вышел - стал птицами. Другие постепенно вымерли.
419 449901
>>49897
Получается так?
r=коэф/широта
Это приближение или что? Почему угол не в какой-нибудь тригонометрии?
420 449904
>>49893
Я кажись обосрался и координаты у меня нихера ни географические изначально, а декартовы - гнуплот их рисует как картинку из атласа, если их как x, y подать. Всё равно не знаю как тогда быть, пытался как на пикче, но и результат тоже херня
421 449905
Двач я знаю что науке до сих пор неясно откуда взялись вирусы но у меня такой вопрос а как вирусы внедряются в живые организмы, каким образом? Ну вот есть например какой-нибудь спид и вполне понятно как он передается между носителями но ведь он же должен был изначально попасть в носителя, был ли такой момент вообще в истории когда живые организмы не были носителями вирусов и в итоге вирусы каким-то образом попали в живые организмы или они были всегда и развивались одновременно с развитием живых организмов?
422 449910
>>49905
Почему не ясно. Просто деградировали из обычных клеток, убрав всё лишнее из неё.
423 449912
Блджад, поясните за торфиз, положительная длина рассеяния означает отталкивание или что? И если да, то почему в этом режиме фермионы слипаются в молекулы?
424 449916
>>49905
Все прокариоты обмениваются информацией через вирусоподобные частицы, вирусы эволюционировали из этих "спор" с ДНК. Нужно это из-за генетического храповика, если не обмениваться работающими генами будешь их терять из-за вредных мутаций, потому такой обмен геномом появился не просто у прокариот, а ещё до появления прокариот. Соответсвенно вирусы-паразиты - по-сути древнейшие формы жизни.
425 449919
>>48550 (OP)
Вы там охуели ли или да? Какого хера обсуждают всякую политику шмолитику, бессмертие и прочие сети, когда такой проект простаивает?! Андронный, андронный... Хуенный! Вот что взяли и проебали почти полностью! Какого еб твою оно не работает??! За десятки лет до этих зарубежных дебилов уже занимались этими коллайдерами, так какого нихера не делаетсяЮ пироды?!!

Как убрать эффект хроматической аберрации со шрифтов, чтобы не было как на последней картинке?
426 449931
>>49830
ну так тут важна не только температура плазмы, но и плотность её излучения и скорость охлаждения. корона солнца имеет температура в миллионы градусов например. но плотность там маленькая и корабль в теории может в таких условиях выжить.
427 449935
>>49919

>хроматической


мой компьютер правой кнопкой - свойства - параметры быстродействия - галочку сглаживание шрифтов
428 449936
>>49827
В итере охлаждение жидким гелием, плюс единовременно там граммы дейтерия. правда это количество плазмы имеет поле эквивалентное тысячамм тонн сил и в итере неебические подпорки у магнитов чтобы не сложиться просто.
429 449959
>>49936
Тут нету никакого противоречия?
Прост отдача тепла идёт от температуры и площади. Если вещества было бы много по массе, ему бы хватило тепла расплавить стенки. А если было бы мало-оно практически мгновенно должно остыть.
430 449978
Авторитетные ученые двача, скажите плес, этот робот - фейк или нет? Мне никак не верится, что в 2018 ИИ может обсуждать философские темы, и я уверен, что на самом деле ее голосом управляет какая-то тетка где-то за стеной. Но тогда почему все говорят, какой у нее охуенный интеллект и какого хуя этому болванчику дали гражданство Саудовской Аравии?
431 449982
>>49978

>Робот использует технологию распознавания речи от Alphabet (материнской компании Google)


>и совершенствуется со временем, становясь умнее.


>Программное обеспечение искусственного интеллекта Софии разработано компанией SingularityNET.


Все вопросы - туда.
432 449999
>>49978
Не фейк. Просто сработал вау-эффект у людей. Если ты даже на том же ютубе пересмотришь как можно большее количество видосов с ней то увидишь что там ничего особенного то и нет.
433 450000
Если ты например с кошкой начнешь разговор, то и она в ответ будет мяукать, а ты такой "ух ты, какая же она умная"
Тот же самый эффект сработал и здесь.
image.png156 Кб, 1280x1024
434 450001
>>49935
Это?
435 450005
На дваче сказали что времени нет, но Эйнштейн сказал время есть, так естьили нет времени в физике?
Просто установили что 14 ,6 миллрд. лет вселенной
436 450006
>>50005
Это не возраст вселенной, а область, которую смогли разглядеть вокруг нашей системы, на 14 ,6 миллиардов световых лет простирается. А какой настоящий возраст вселенной - никто не знает.
437 450007
>>50006
Лалка
438 450009
Никаких прорывов в аккумуляторах не будет?
Литий как понимаю это топ и максимум, иначе с ним бы не работали, а работали с тем что топ.
А других элементов не завезли.
Будут только понемногу допиливать технологию, по чуть-чуть добавляя ёмкости? До.. до скольки? Какой теоретический максимум литиевых аккумуляторов?
439 450013
Если нарколыге предоставить бесконечно героина (предострегши от передоза), будет ли момент полного угасания кайфа и наступления толерантности, депрессии и прочей хуеты?
440 450014
>>50009
Будет, есть куда стремится.
Начнем с того, что в природе есть почти прямые преобразователи химической энергии в электрическую. Они работают в наших клетках, где различные фермеры через различные циклы создают на мембране потенциал. И работают таки с очень высоким КПД. Теоретически возможно создать такие структуры, где процесс возможен был обратим, этакая глюкозная батарейка. Но у нас нет таких технологический чтоб делать молекулярные машины наподобие естественных биологический.
Второе, каждое вещество обладает некоторой внутренней энергий. Есть дохуя веществ, которые условно стабильны при разных уровнях внутренней энергии, но как всякая система она стремится к минимуму внутренний энергии, что выражается в возможности высвобождение излишка энергии. Но тут проблема в подборе этих веществ и преобразование энергии и эта довольно сложная задача, которая сводится прям к нанотехнологиям - манипуляцию с отдельных атомами и построению хитрых кристаллических решеток.
Литий-ионные аккумуляторы же одна из самых отработанных и масштабируемых технологий на данный момент, поэтому и используют ее массово. Энергетическая ёмкость только ограничена качество электролита и типом электрохимической реакции, но все равно маленькая.
441 450016
>>49900
>>49894

Да что вы несёте, люди? Я точно на наукаче? Какой к чёрту тупиковый вид? Это надотряд, НАДОТРЯД, он не мог быть весь тупиковым, как блядь не могут быть тупиковыми птицы, отдельные виды, безусловно, но не весь надотряд.

>динозавры не вымерли, а эволюционировали в птиц. а сами по себе тупиковой ветвю были. слишком большие и зачем такое тело кормить.



Я так понимаю, что ты думаешь что ВСЕ мелкие виды эволюционировали в птиц, а все крупные вымерли. Но это не так. Далеко не все мелкие виды были дромеозавридами, но вымерли почему-то все.

>На то время так же и цветочные растения активно плодиться начали, которые динозавры есть не могли.



А можно пруф? Тем более, папоротники, хвощи и хвойные никуда не делись, этот фактор мог уменьшить популяцию, но никак не уничтожить целый надотряд. К тому же, даже если крупные растительноядные от этого вымерли, обрушили трофическую пирамиду, что привело к вымиранию крупного и среднего размерного класса хищников, были ещё насекомоядные и мелкие хищники, которые жрали в том числе и млекопитающих, которые отлично продолжали себя чувствовать, и нет оснований полагать, что были в своей нише успешнее. А как же птерозавры и морские ящеры? Эти к цветочным вообще никакого отношения не имеют.

>Так что вымерли они скорее всего из-за комплекса проблем



Это-то безусловно.

> и в целом тупиковым видом были.



Об этом выше.

>Кто из тупика вышел - стал птицами.



Птицами стало одно семейство, всего лишь. По какой причине вымерли все остальные дейнонихозаврид? У них не было принципиальных морфологических отличий.

>Другие постепенно вымерли.



Опять же, безусловно, но вопрос ведь почему?

О, и кстати, вы так говорите, что вымирание было постепенным, как будто раньше считалось что метеорит их прямо на месте укокошил. Насколько я знаю, ни одно из Великих вымирание не было даже относительно мгновенным.
441 450016
>>49900
>>49894

Да что вы несёте, люди? Я точно на наукаче? Какой к чёрту тупиковый вид? Это надотряд, НАДОТРЯД, он не мог быть весь тупиковым, как блядь не могут быть тупиковыми птицы, отдельные виды, безусловно, но не весь надотряд.

>динозавры не вымерли, а эволюционировали в птиц. а сами по себе тупиковой ветвю были. слишком большие и зачем такое тело кормить.



Я так понимаю, что ты думаешь что ВСЕ мелкие виды эволюционировали в птиц, а все крупные вымерли. Но это не так. Далеко не все мелкие виды были дромеозавридами, но вымерли почему-то все.

>На то время так же и цветочные растения активно плодиться начали, которые динозавры есть не могли.



А можно пруф? Тем более, папоротники, хвощи и хвойные никуда не делись, этот фактор мог уменьшить популяцию, но никак не уничтожить целый надотряд. К тому же, даже если крупные растительноядные от этого вымерли, обрушили трофическую пирамиду, что привело к вымиранию крупного и среднего размерного класса хищников, были ещё насекомоядные и мелкие хищники, которые жрали в том числе и млекопитающих, которые отлично продолжали себя чувствовать, и нет оснований полагать, что были в своей нише успешнее. А как же птерозавры и морские ящеры? Эти к цветочным вообще никакого отношения не имеют.

>Так что вымерли они скорее всего из-за комплекса проблем



Это-то безусловно.

> и в целом тупиковым видом были.



Об этом выше.

>Кто из тупика вышел - стал птицами.



Птицами стало одно семейство, всего лишь. По какой причине вымерли все остальные дейнонихозаврид? У них не было принципиальных морфологических отличий.

>Другие постепенно вымерли.



Опять же, безусловно, но вопрос ведь почему?

О, и кстати, вы так говорите, что вымирание было постепенным, как будто раньше считалось что метеорит их прямо на месте укокошил. Насколько я знаю, ни одно из Великих вымирание не было даже относительно мгновенным.
442 450021
Какие следствия вытекают или могут вытекать из того, что скорость распространения гравитации согласно измерениям равна скорости света вплоть до 15-го знака после запятой, или вообще полностью равна ей?
443 450022
>>50021
ОТО очередной раз не обосралось.
444 450023
>>50022
Хех, я оценил, но думаю ты понял о чём я.

Типа два совершенно разных поля якобы, и тут бац, можно сказать абсолютно одинаковое свойство у обоих. Это ж неебацца какое совпадение. Так не бывает. Что-то за этим скрывается.
445 450024
>>50023

>Что-то за этим скрывается


Геометрия пространства-времени. Где скорость света предельная скорость.
446 450025
>>50024

>Где скорость света предельная скорость.


Без проблем.
Почему не меньше?
447 450026
>>50025
Потому что гравитон безмассовый иначе гравитация была бы локальной.
448 450027
>>50025
Никто не ответит тебе "почему С именно С а не на один метр больше или меньше". Возможно есть какие-то уникальные люди, которые пытаются из теории струн выводить константы, но это философия уже.
449 450029
>>50026
А, точно, вспомнил, после того как прочитал что ты написал, один видосик на ютубе, про безмассовые частицы, что они не могут не двигаться на скорости света.

>гравитон


Хм, ладно фотон передаёт энергию-импульс. Но гравитон..если он есть что он мог бы передавать и как? Отрицательное "толкание"?

Так, стоп, масса же искривляет пространство, и материя просто падает в область наибольшего искривления. И скорость "распространение гравитации" это просто скорость распространения искривления пространства.
450 450030
>>50027
Я про гравитации писал.
451 450032
>>50007
ну не знаю я про время, я про возраст немного знаю.
Гриффин в шкафу.жпг
452 450033
>>50016

>Я так понимаю, что ты думаешь что ВСЕ мелкие виды эволюционировали в птиц, а все крупные вымерли. Но это не так. Далеко не все мелкие виды были дромеозавридами, но вымерли почему-то все.


>О, и кстати, вы так говорите, что вымирание было постепенным, как будто раньше считалось что метеорит их прямо на месте укокошил. Насколько я знаю, ни одно из Великих вымирание не было даже относительно мгновенным.


Вымирание динозавров ничем не отличалось по темпу вообще от среднестатистического вымирания видов. Просто это у них не компенсировалось образованием новых видов.
Вот вымер один вид кошака, но в то же время образовался или откололся другой.
В случае динозавров вид вымирал и на замену ему другой не приходил. А большие могли страдать не только от проблем с пропитанием, но и от постоянных набегов мелких хищников на их детишек. Большие же животные плодятся долго и по мало, поэтому для них это критический урон.
453 450035
>>50033

Да чего ты зациклился на больших-то? Динозавры - надкласс, это огромная куча жиотных самых разных видов. Даже полное вымирание мегафауны ничего не значит для всего надкласса.

>А большие могли страдать не только от проблем с пропитанием, но и от постоянных набегов мелких хищников на их детишек.



Я вроде читал, что эта гипотеза признана несостоятельной. Но тут надо у нормально шарящего в теме анона спросить, я с ходу пруфов не дам.

Да и в любом случае, ниша мелких хищников была занята другими динозаврами, там млекопитающие не доминировали в конце мела.
454 450037
Могут ли тяжелые металлы выделяться из гидросферы в атмосферу? Если вообще могут, то возможно ли это при естественном испарении?
455 450038
>>50037
Ноуп, только если это не гейзер или прочая вулканическая залупа.
456 450041
>>50038
Огромное спасибо.
457 450044
>>48550 (OP)
Если первая реакция протон-протонного цикла идет 10 млрд лет, как зажглись первые звёзды?
458 450046
>>50044

>Если первая реакция протон-протонного цикла идет 10 млрд лет, как зажглись первые звёзды?


Если это так. То они не зажглись.
459 450047
>>50046
Тогда по какому механизму зажглись первые звёзды?
460 450048
>>50047
По протон-протонному циклу.
462 450050
>>50049
Как думаешь, что эти цифры под стрелками обозначают?
463 450051
>>50050
Шесть миллиардов лет на первую стадию, самая старая звезда считаеть всего на поллмилиарда лет моложе вселенной. Как она зажглась?
464 450052
>>50051

>Шесть миллиардов лет на первую стадию


Как ты понимаешь, эту фразу?
Куча протонов ждет ровно шесть миллиардов лет, после чего ВСЕ резко превращается в дипротон?
465 450053
>>50052
Cреднее время за которое пара протонов объединяется
466 450054
Я вообще не уверен туда ли я зашел, петушок. Но меня мучает вопрос: если теория эволюции верна, то почему никто кроме людей не эволюционировал до такого уровня и почему при этом остались обезьяны, которые не стали такими же развитыми. Почему не появилось летающих и водоплавающих видов, развившихся по интеллекту до уровня людей, почему моря не населены высокоразвитыми цивилизациями сверхинтеллектуальных рыб, а в небе не летают суперумные высокотехнологические птичьи народы? Почему только люди?
467 450055
>>50053
Это и есть ответ на твой вопрос. Время полураспада урана например 4 510 000 000 лет, а среднее время распада 6 506 000 000 лет.
Аналогично и тут. Это просто показатель того, как медленно идет эта реакция, но она идет.
468 450056
>>50054
А какая движущая сила у эволюции дельфинов или птиц? Зачем им становиться умнее?
А обезьяны не "остались", они отпочковались от нашего общего предка. Они не "родители" они "братья".
469 450057
>>50054

>если теория эволюции верна, то почему никто кроме людей не эволюционировал до такого уровня


Потому что развитый мозг и язык - это очень дорогой апгрейд, польза от которого проявляется не сразу. Человеки за него десятки тысяч лет расплачивались достаточно днищным, полупадальщическим образом жизни, подъедая мяско за львами в саванне, - а также обросли хуевой кучей физических проблем, вроде слишком крупной головы, мешающей при рождении; хуёвого позвоночника от прямохождения; шизофрении от языка.

>почему при этом остались обезьяны, которые не стали такими же развитыми


Всякие шимпанзе - не "остались". Это другая ветвь, которая эволюционировала другим путём от общего с человеком предка.
Просто кто-то вкладывался в мозг, а кто-то в бицуху.

>Почему не появилось летающих и водоплавающих видов


Рук нет.

>а в небе не летают суперумные высокотехнологические птичьи народы?


Птицам для выживания нужно летать => маленькие размеры => мелкий мозг.
А так-то врановые и попугаи по интеллекту равны или даже умнее шимпанзе (а у шимпанзе интеллект на уровне 2-летнего человеческого ребёнка).
470 450058
>>50044
Это среднее время, какие-то раньше, какие-то позже, а их в звезде супер дохуя, и в среднем она стабильно и мощно горит.
471 450060
>>50058
Учитывая 10 млрд... лучше сказать мощно тлеет.
472 450062
>>50049
Что значат эти реакции подробно?
Всымсле, при каких это условиях? Два протона один на один, или много, в каком объёме замкнуты, при каком давлении, при какой температуре?
И почему есть это среднее время реакции, как я понимаю, при любых условиях, почему нет гарантированной реакции?
473 450063
>>50062

> в каком объёме замкнуты


Внутренняя (<.3R) Cолнца

>при каком давлении


~100ГПА

>при какой температуре


4kk K
474 450064
>>50063
А, т.е. это для солнца расписано, спасибо.
476 450066
>>50065
ок, я завтра посмотрю это целиком.
477 450067
Что определяет является ли элемент газом при н.у.?
478 450074
>>50067
Целая куча факторов. Есть закономерности, если посмотришь на таблицу Менделеева, (в которой температуры кипения отмечены). Но сказать как отрезать нельзя. Всё в физику упирается сложную.
479 450079
>>50060
Мощно тлеют красные карлики. Пылают голубые гиганты. А вот жёлтые карлики вполне себе нормально горят.
Хотя с научной точки зрения они и не горят. Это совсем другая реакция.
Они находятся в перманентном термоядерном взрыве, но гравитация не даёт взрыву всё это раскидать по вселенной и удерживает их в компактном шарике.
480 450082
Человеческий интеллект биологически детерминирован?
481 450083
>>50082
Нет, конечно. Вот, например, откуда перелётные птицы знают, где находится север и юг? Душа им говорит.
482 450085
>>50082
скорее всего да, но рассчитать его не представляется возможным. даже десятую долю от 1% от него.
483 450087
>>50074
Спасибо за ответ.
484 450088
>>50037
Конечно могут. У любого вещества есть парциальное давление, которое зависит от массы атома/молекулы и температуры.
485 450090
>>50067
Упирается в электронные оболочки и атомную массу. Электронные оболочки определяют - могут ли иметь связи атомы. В благородных газах оболочки полные - поэтому никаких связей быть не может - поэтому газ. В щелочных металлах связь металлическая, так как внешние электроны оказываются в зоне проводимости уровень ферми в разрешенной зоне, поэтому выгодней формировать кристалл, чем бинарные молекулы. А в галогенах зона проводимости отделена широкой запрещенной зоной, поэтому все что остается - формировать бинарные молекулы с ковалентной связью. Если валентность достаточно велика, то ковалентных связей может хватить, чтобы образовать связи в объемный кристалл, либо сформировать листы, которые будут держаться силами Ван-Дер-Ваальса или пи-связями. Если вещество может образовать кристалл, то оно его образует, это выгодно по энергии. Не все наверное так однозначно можно объяснить, но примерно как-то так.
486 450091
Еще со школы беспокоит один вопрос: чему равно ноль делить на ноль. Я безусловно врубаюсь что на ноль делить нельзя, но несмотря на это этот вопрос просто не вылетает из моей головы
487 450092
>>50091
Деление на ноль в строгом математическом понимании смысла не имеет.
В теории пределов, если одна последовательность стремится к нулю, то деление конечного числа на эту последовательность дает предел бесконечность, то есть результирующая последовательность стремится к бесконечности.
Если есть 2 последовательности, и они стремятся к нулю и делятся друг на друга, то это называется неопределенностью типа 0/0. Только в таком понимании можно делить ноль на ноль. Результат зависит от закона по которому описываются эти последовательности. Например если первая последовательность 1/n, а вторая 1/n^2, то первое делить на второе при n стремящемся к бесконечности даст бесконечность, наоборот - даст ноль. Результат может быть вообще любым - зависит от последовательностей, может быть какое-то конечное число.
488 450093
>>50092
Спасибо
489 450095
>>50091
Добавлю к ответу анона >>50092 (и воздержусь от комментариев о математической строгости его ответа, так и ответа ниже):
Числа, понимаемые "интуитивно", описываются математическими структурами (кольца, поля). Можно определить новую структуру, в которой разрешено деление на ноль (другими словами, всякий элемент обратим), но из-за этого элементы будут обладать "странными" или "неудобными" сврйствами (например, в обычной компактификации вещественной прямой новый элемент "не похож" на нормальные числа).
Такие структуры используются и полезны - например, проективные пространства вроде проективной вещественной прямой и сферы Римана. Есть понятие алгебраического колеса - алгебры, в которой каждый элемент обратим.
490 450096
>>48550 (OP)
Такой вопрос. Проводник отличается от изолятора тем что в нём много свободных электронов. Батарейка создаёт две силы, одна притягивает электроны, а другая отталкивает.
1. Если к материалу подключить сборщик электронов, почему свободные электроны не перетекут в сборщик? Тогда проводник должен стать изолятором? Золото хороший проводник, то во что оно превратится после такого?
Так как золото не меняется, то я не знаю что думать.
491 450097
>>48550 (OP)
Какое пиротехническое устройство может поднять в воздух пару килограмм нной субстанции на высоту в 2-2.5 метра, после чего взорваться и расшвырять оную по кругу радиусом в 1.5-2 метра?
492 450110
>>50097
иди в военач спроси. там тебе эксперты по боеприпасам объемного взрыва насуют ссылок на прыгающую бетти
15350281218650.png120 Кб, 736x736
493 450111
>>50096

>Проводник отличается от изолятора тем что в нём много свободных электронов.


откуда берутся свободные электроны, если все атомы проводника друг с другом связаны химическими связями?
494 450113
>>50014

>И работают таки с очень высоким КПД


хз... хз... Смотря что именно ты считаешь под КПД.
495 450114
>>50029

>Но гравитон..если он есть что он мог бы передавать и как? Отрицательное "толкание"?


Точно так же, энергию-импульс.

Странно что тебя смутил гравитон, а фотон не смутил. Ведь заряды притягиваются.
496 450115
>>50082
Пенроуз целую книгу написал, где рассуждал об этом.
Тени разума называется. Почитай, он там обсасывает со всех сторон этот вопрос.
497 450116
>>50088
Если парциальное давление меньше числа Авогадро, то можно сказать что нету их в атмосфере.
498 450117
>>50096
Есть аналогия, которая худо бедно описывает какой-то процесс. Но брать её и пытаться описать этой аналогией иной процесс, приводит к таким вот парадоксам как у тебя.
499 450119
Почему у почти всех животных есть табу на каннибализм? Это связано с эмпатией, или есть какие-то чисто биологические причины типа прионов?
500 450120
Можно ли в формуле Кардано привести вещественные корни, которые содержат мнимую единицу к их истинно вещественному виду (то что называется неприводимый случай)? Просто еще когда в универе учился мне препод по матану рассказывал, что он якобы смог так сделать. Правда так и не показал как. Или он пиздел?
501 450122
>>50116
не понял как можно парциальное давление сравнивать с числом Авогадро, это величины разных размерностей. Вопрос не в количестве, а сигнатурном определении. Если в океане растворено золото в виде суспензии, то и в атмосфере оно есть. Сколько его на всю планету - миллиграммы или пару тонн, уже другой вопрос.
502 450123
>>50122

>это величины разных размерностей


Не всегда, порой меньше даже. Посчитай размерность давления NaCl над NaCl
503 450124
>>50123
Что меньше? Давление измеряется в Паскалях, число Авогадро измеряется в штуках. Что больше - 2 километра пути или 5 килограмм? 5 яблок или 220 вольт в сети? Понимаешь, что такое размерность?
504 450125
>>50096
Когда ты из куска металла вытягиваешь свободные электроны, он заряжается. Чтобы убрать все свободные электроны, потребуется такое огромное напряжение, что практически это не осуществимо. Естественно выкачав все электроны, кулоновские силы просто развалят решетку на отдельные ионы.
>>50111
Они связаны свободным электронным газом, который удерживает ионы вместе кулоновским притяжением. То что орбитали в атомах не полны, по принципу Паули даёт возможность электронам свободно перемещаться по узлам решетки
505 450126
>>50124
Парциальное давление непрерывно связано с мольной долей. Ку!
506 450127
>>50126
но это не одно и то же. Ток связан с напряжением, это не дает мне право сравнивать вольты и амперы.
507 450128
>>50127
Давай без аналогий глупых. Парциальное давление очень четко дает тебе концентрацию в моль/м^3
Если у тебя парциальное давление ниже блять чем 10^-23, а для металлов и ионов (тут вообще пушка), у тебя в -30 -40 спокойно может получиться порядок.
То и концентрация у тебя теряет физический смысл.
Вопрос то был про "естественное испарение". Понятно, что всякие аэрозоли и другие процессы выбрасывают в атмосферу всю эту грязь. Но аргументировать это тем, что у всего есть парциальное давление, стремно довольно.
508 450129
>>50128
Два пункта.
Во первых.

>Посчитай размерность давления NaCl над NaCl


Размерность это единицы, в которых измеряется величина. Для давления неважно какого вещества эта единица одна - Паскаль.
Во вторых.
Я говорю про возможность сублимации любого элемента. Сколько его там - это надо считать. Может 1 атом на всю планету. Может и меньше, а может и больше. Это статистическая величина, которая определяет вероятность перехода из одного состояния в другое, согласно распределениям термодинамики.
На этом предлагаю закончить спор.
509 450135
>>50125
Для ЭДС обязательно двустороняя батарейка, которая с 1 стороны подталкивает, а с другой притягивает? Если присоединить только с недостатком электронов, она вытянет немного электронов из провода?
510 450142
>>50113
Все просто, под КПД подразумеваю отношение работы на захват электрона/освобождения протона к химической энергии реакции.
511 450144
>>50119
Канибализм у всех животных наблюдается изредка. Даже сегодня. Просто после такого группа это обычно перестаёт существовать. Ведь едят в большей мере детей же.
512 450145
>>50144
Бля, неужели все так просто? Я-то думал.
513 450146
>>50145
парочку каннибалов просто бытро уничтожают эту группу. животные же. у них мозг не так развит, что бы превентивно защищаться. Поэтому вот живёт среди них каннибал спокойно, они то принимают его за своего, а он их постепенно съедает. Редко это, конечно, ибо это отклонение явное. Но у животных уже давно развился механизм свой-чужой. а без него не выжить просто виду.
514 450148
>>50145
Ну и чужих даже своего вида тоже едят часто. Но они воспринимаются именно как чужие. Какое то дикое племя легко могло называть каких то хищников и жителей других племён как одинаковую угрозу а так же добычу.

Собаки и прочие животные так же могут легко съесть других собак. Но едят то понятно дело слабых, на равного волк нападать не будет. Поэтому среди хищников такое редко в дикой природе.
515 450154
>>50114

>Точно так же, энергию-импульс.


Как можно энергией-импульсом передать притягивание?>Странно что тебя смутил гравитон, а фотон не смутил. Ведь заряды притягиваются.
Тут вообще не понял что ты этим хотел сказать.
516 450155
>>50135
Да, таким образом, чтобы сравнять потенциалы. Учитывая ничтожную емкость провода вытянет где-то 10^-9 кулона (около 10 млн электронов). Но и в самой батарее поле меняется, нельзя заземлить один полюс и разрядить "с одной стороны"
517 450158
>>50155
Спасибо за ответы.
518 450164
Поел вчера пюрешки с котлеткой и сразу выпил 500мл воды, просто захотелось попить, батя начал бухтеть, мол, вода проталкивает пищу в кишечник и она там начинает гнить а не перевариваться, ну я погуглил и так и не смог разобраться. Кто-то пишет про плохое усвоение жиров, кто-то про NCl, а доктора считают что все заебись и единственное не надо пить сразу 2 литра иначе будет опьянение водой. Так пить или не пить?
519 450166
>>50164

>Так пить или не пить?


сразу видно нерусского человека.
1463521588068.jpg27 Кб, 268x268
520 450169
Ботаны, объясните мне, почему в РФ такой высокий уровень убийств - 10.82 на 100 тыс. населения? Это самый высокий показатель для стран с европеоидным населением, выше только африканские страны и южноамериканские страны с высоким процентом метисизации (и то далеко не все).
Вна той же Украине этот показатель - 6.34 - что тоже довольно-таки пиздецово и по-африкански/латиноамерикански. Но всё равно почти в 2 раза ниже РФ, при том что Украина в 3 раза беднее.
Казахстан - 4.81, Белоруссия - 3.58, прочие обломки СССР - ещё лучше.
В странах Европы, даже беднейших и пост-коммунистических, убийства случаются в 10 раз реже, чем в РФ
521 450175
Поясните, по какому принципу действуют маскираторы речи типа https://www.youtube.com/watch?v=xvaqHbZqQ8A ?
Ясно, что при записи подкинутым в помещение жучком ничего разобрать нельзя будет (если тембр голосов маскиратора выбран правильно). Но почему тогда при таком включенном девайсе собеседники в комнате понимают друг друга?

Inb4: слух бинаурален и мозг автоматически вычленит речь собеседника (если только рот собеседника и маскиратор не расположены в одной точке). Значит ли это, что против таких маскираторов могут помочь жучки с, кхм, опцией бинауральности (например, в простейшем случае имеющие два разнесенных в пространстве микрофона, по запаздыванию одного и того же внешнего звука на которых можно определить его положение)?
522 450188
>>50169
в центральной россии где то от 4 до 8 по разным областям.
а уж где то в туве и выше африканского поднимаетсся.
главная причина в алкоголизме, ибо основные убийства у нас бытовуха, а не делёж территорий бандами.
523 450191
>>50188
Так в Туве живет всего 320 тыс. человек. Кстати, ещё одна загадка: почему российские северо-восточные азиаты настолько буйные, они во всем мире самая смирная группа этносов.
Это не ответ. Алкоголь есть везде, "культура потребления" в сранах б.СССР ровно такая же - а убийств меньше в разы.
524 450194
>>50191
Убийств там столько же, написали же тебе. В центральной России различий по убийствам между Украиной и Белоруссией нет. Где то побольше, где то поменьше, в зависимости от депрессивности области. Сейчас где то 5-6 на 100000.

Ну а причины такого колличества убийств в сибири и дальше уж отдельный вопрос. На счёт этих азиатов, то проблема возможно в том, что СССР по сути и уничтожил их уклад жизни и культуру. Поэтому живут в коммиблоках, бухают, а от алкоголя у них башку больше срывает, вот и валят друг друга массово. Плюс туда и так постоянно всяких уголовников ссылали, поэтому всякая ауешная культура там сильнее.
525 450195
>>50191
А то, что ты берёшь 10 на 100000 это вообще данные за 2011-2012, кажется недавно, но убийства у нас каждый год стабильно падают и довольно быстро. За последний год по России в целом, с учётом Тувы и прочего 7 где то.
526 450222
а я опять прихожу сюда после просмотра очередного науч-поп видосика на ютубе
Так стоп, если гравитации как силы нет, а есть только движение по искривлённому пространству-времени, то почему говорят что свет испытывает красное смещение, "взлетая" с массивных объектов?(со всех впринципе, но при взлёте с массивных это хоть как-то заметно)
527 450223
>>50119
Просто все кто жрали себя вымерли, в природе всё так работает, очень просто.
Если что-то есть-то оно не вымерло, а если чего-то нет-то оно вымерло.
528 450224
>>50129
>>50128
Т.е. рано или поздно, именно естественно испарятся даже свободно летающие в космосе каменные глыбы?
529 450225
>>50194

>В центральной России различий по убийствам между Украиной и Белоруссией нет.


штоблядь
Это на какой вопрос ответ? Ладно, что он фактически неверный, но к чему это вообще?
>>50195
10.82 это за 2016 год.
279718358293545539337229097792620711280699n.jpg70 Кб, 945x960
530 450226
>>48550 (OP)
Спрошу здесь, так как больше не нашел где.
Когда-то в журнале "Наука и Жизнь" была статья/задачка, про клинопись. Там приводилась запись клинописи и рассказывалось как немец Гротефенд расшифровал ее. Ну и предлагалось повторить подвиг немца и тоже расшифровать надпись. В общем ничего особо сложного.
Но я напрочь забыл номер журнала. Вдруг здесь кто-то вспомнить эту статью? Ну или не из журнала но похожую. Что-бы и метод был описан и сама клинопись, та самая была. Просто сколько не искал везде или без самой надписи или что-то левое чисто для иллюстрации кидают.
531 450228
>>50224
Да. Об этом в учебниках по физике пишут. Но этот процесс фактически бесконечен. А вот то что атмосферы у планет улетают, это вообще норма.
532 450229
>>50225

>Это на какой вопрос ответ? Ладно, что он фактически неверный, но к чему это вообще?


К тому, что именно центральная Россия больше всего похожа на эти страны. Там убийств примерно 6 на 100000. В более депрессивных поболее этого, в менее депрессивных областях по менее.

Ну и не забывай, что Россия страна куда больше, и что самое главное тут куда больше мигрантов. А на них убийства куда чаще приходится.

А из Украины и Беларуссии молодое население наоборот уезжает, что несколько и положительно на преступности сказывается.
533 450231
Перекооот пилите
я не умею((
534 450238
>>50065

У него тут есть ряд моментов, с которыми я принципиально не согласен. Например он говорит о невозможности развития соответствующего мозга у драмеозаврид, в силу их горизонтальной ориентации относительно земли, мол будет перевешивать голова с большим мозгом. Это полный бред. Прямые потомки - птицы семейства фороракосовых имели огромную голову, чуть ли не в трет массы тела, и ничего, никаких проблем. У них там конечно не мозг был, но морфология им совершенно не мешала.

>>50057
>>50056

А сюда добавлю, что следы цивилизации - сохраняются намного хуже на относительно большом геологическом периоде времени, чем удачно законсервированные останки. С учётом ничтожно малого представления о видовом многообразии палеонтологией, я бы не стал заявлять о принципиальной невозможности существования разумных видов и даже цивилизаций в отдалённом геологическом прошлом.
535 450252
>>50238
У австралопитеков мозг стал расти лишь тогда, когда у них стала уменьшаться челюсть.
Да и огомной она может казаться лишь тебе. Добавь туда 1.5кг мозга и она станет и у них в три раза больше.
600px-Paraphysornisbrasiliensis1.jpg185 Кб, 600x900
536 450253
>>50252
Череп то большой. Но в итоге 3/4 занимает сам клюв. А мозга там явно совсем кот наплакал. Да и в ходе эволюции у них череп лишь уменьшался, что явно намекает на то, что такое таскать не так уж и просто.

Ну и мозга совсем мало. Пока и у других его было мало могли бегать с такой тущей. Как другие поумнели, то тут же вымерли.
537 450254
>>50253

Да я не спорю. Но бошка тяжелая. А Дробышевский утверждает, что морфология не позволит горизонтально ориентированным бипедальным существам носить тяжелую голову, вот наглядный пример, что позволяет при минимальны отличиях.

>>50252
Конечно, но при мозге у них бы не было такого клюва. Я говорю про то, что голова весом почти в треть от всего веса особи - не проблема.
538 450255
>>50254
Там речь то не о том, что не могут тяжелую, а не могут тяжелее и при этом эффективнее.

>Конечно, но при мозге у них бы не было такого клюва.


А с чего бы ему пропадать? У австралопитеков челюсть начала меняться не из-за роста мозга, а из-за смены образа жизни. Питаться другой пищей стали и крепкие челюсти им стали в ненадобности. Это после уменьшило нагрузку на голову и позволило им резко нарастить ещё мозга.

Но надо понимать, что у них и уже мозга было немало. А относительной этих птичек они вообще гениями были.
14711431462462.jpg30 Кб, 578x188
539 450257
>>50037
Даже алмаз и даже Аллах испаряются даже при абсолютном нуле. Смекаешь? Вопрос в продолжительности. Тяжёлые металлы делают это не особо охотно. С другой стороны, ты можешь провести простой эксперимент -- разлить ртуть у себя дома (если у тебя частный дом и ты живёшь в нём один) под окошком с солнечными лучиками, и засечь время, когда тебе захочется обратиться к пульмонологу.
540 450280
Каким образом электроны в сети отдают энергию приборам? Типа толкают механизм и всё медленнее движутся при приближении к нулевому проводу? И в конце они типа уставшие, но куда они дальше деваются? А если между прибором и нулём подсоединить себя, то вы с прибором поделите падение напряжения, допустим, поровну, но как электроны узнают, что теперь на прибор надо тратить не всю, а половину энергии? Получается, сопротивление экранирует поле или чё? А ещё, часто пишут, что дешевые бп гадят сеть пульсациями, но куда они их бросают - в ноль, землю или фазу? И могут ли они ударить током, пульсации эти?
sage 541 450281
>>50280

> электроны в сети отдают энергию приборам?


Переносчик энергии всё же не электрон, а эл.м. поле. Электроны лишь среда, которая помогает. Короче это всё сложно, так как можно смотреть с любой стороны какой хочешь на эти процессы.
542 450299
>>50257
оу, оу, про абсолютный ноль откуда инфа? Выигрыш в энтропии зануляется, распределение съябывает, сублимации вроде как бы и не происходит, не?
543 450300
>>50281

>Электроны лишь среда, которая помогает.


Но среда же электромагнитное поле, в которой существуют электронны. Из которой состоят, являясь возмущением его.
15379820080490.jpg166 Кб, 474x631
544 450302
>>50299
Так и думал, что здесь озадачатся.
В дело здесь вступает квантмех.
Если хочешь реально вгрызться, сначала прогугли отрицательные абсолютные температуры.

> Выигрыш в энтропии зануляется


А закон возрастания энтропии в изолированной системе, кстати, хоть и наблюдается повсеместно и практически, но на действительно больших промежутках времени, например, не имеет характера абсолютного закона.
545 450303
>>50255

Ты посты-то читаешь, чуть раньше предыдущего? Речь не про фароракосовых, а про дромеозаврид. У некоторых видов из которых, был судя по всему очень развитый мозг. Безусловно, для дальнейшего развития им бы вполне возможно пришлось залезть на деревья и спуститься с них, но древолазающие виды тоже известны, а значит это не невозможно. Но Дробышевский пишет, что никаких вариантов, горизонтальная ориентация относительно земли рубит всё на корню, а я говорю, что очевидно морфология - не железное ограничение в данном случае.
546 450318
Какой нужно толщины взять титан чтобы он выдержал ядерный взрыв, скажем, 10 килотонн (в центре взрыва, на расстоянии 100м и 1км)
547 450319
>>50318
Это вопрос из разряда "посчитайте за меня", некультурно такое спрашивать.
548 450321
>>50319
Так вроде тред для таких вопросов и создан (я не знаю как рассчитать
549 450324
>>50303
Он в одном из видео своих и сам говорил, что в теории они могли поумнеть дальше, если бы не было млекопитающих и хищников много вокруг них.
А так они всё равно не смогли соперничать с млекопитающими в одних и тех же задачах, в добавок от хищников бы их более развитых мозг всё равно бы не спасал. Поэтому и растить бы они его не стали. Куда профитнее растить клюв и крепкие ноги, что бы бегать и бить ими
550 450327
>>50321
Нет.
Так и спроси как рассчитать, а не сразу результат расчёта спрашивай.

Для начала определись что ты имеешь ввиду под "выдержит". Не проплавится ли, не разорвётся ли давлением(при этом нужно думать над замкнутым объёмом), не разорвётся ли ударной волной(тут надо думать лежит ли он на чём-то и чём, натянут или положен на опоры, какие и как) или сочетанием этих факторов.
Или вообще может принять за "невыдерживание" не разрыв, а деформацию, насколько сильную..
И т.д.
551 450333
>>48550 (OP)
Я вот тут почитывал вики про смарт карты, написано, что у каждой карточки есть свой процессор и память. Но конкретные цифры не приводят. Мне вот интересно, можно организовав питание и вход/выход сделать из смарт карты вычислительную машину? не совсем по науке вопрос, соре
552 450354
>>50333

>есть процессор и память


>можно сделать вычислительную машину?


Ну вопрос просто пиздец же.
Очевидно что если там это есть, то да, а если нет, то нет.
553 450357
>>50324

Ты про крупных нелетающих? Не, не думаю. Сверхспециализированный облигатный хищник, мне видится крайне слабым кандидатом на эту роль. Тем более во время своего господства они были вершиной трофической пирамиды, таким умнеть смысла нет. А как появились крупные кошачие, так они и вымерли, не выдержав конкуренции. Плюс нет нормального манипулятора, сколько им ещё передние лапы растить заново, а главное, зачем? Так что в потенциальную разумность этих ребят совсем слабо верится.
554 450369
Короче , такой вопрос:есть ли научно доказанный и проверенный способ увеличения полового члена мужчины?Чёт в инете гуглил,выдавало только какие-то обсосные китайские крема и тайский свечки российского производства.Пж, очень надо.
555 450381
>>50354
Жалко всякие кулибины не замандят видосик на трубе. Было бы забавно глянуть на комп, где использованные банковские карты, как картриджи на денди юзают.
Я просто в теме совершенно не але, понятия не имею, переписываемая ли вообще там память. Вроде слышал что даже операционная система стоит, но можно ли эту карту форматировать.
Мож там для вычислений еще что-то надо.
На что вообще такой девайс будет способен? Калькулятор на нем сделать получится? Змейку как на старой нокии?
556 450387
>>50369
Нет, смирись и найди себе другие развлечения, займись наукой.
Хотя вот пластичекая операция, пересадка, в том числе и выращенного искусвенно, и генная терапия должны справиться
557 450388
>>50381

>На что вообще такой девайс будет способен? Калькулятор на нем сделать получится? Змейку как на старой нокии?


Ищешь реально там где-то стоит микросхема.
Ищешь на неё даташит.
Узнаёшь из него её характеристики и узнаёшь "на что она способна"

Только в этом твоём сообщении понял что ты про банковские карты.
Тебе срут в голову, какая нахуй в них микросхема и нахуя? Этой карте нужно просто хранить номер твоего счёта, на него заходит и все операции по нему совершает уже устройствотв которое ты всунул какрту и которому ты дал считать номер своего счёта.
И, какая нахер микросхема? Её работа подразумевает питание. Ты часто свою кредитку подзаряжаешь?
558 450395
>>50388
Я тоже думал, что на банковских просто записанный номер, а симка просто хуевая флешка и все.
Но что-то сейчас засомневался, вдруг при каждом снятии бабок че-то туда пишется (какие-то отметки, или хуй знает что).
На счет процика тоже не понятно, может он юзается для шифрования - типо защита от взлома - своя ось свой проц и т.д.
Короче полистаю литру на которую ссылается вики, как я понял стандартов смарт карт очень много.
559 450396
>>50388
>>50395
Да, автофикс. На счет питания, ясен хуй напряжение на чип подает считыватель. С пей пасса, используется спиралька несколько витков, которая индуцирует токи, для всей этой хуйни, такое даже в одноразовых пропусках в метро есть. Иначе бы она сопссна не считала бы.
560 450409
>>50396

>для всей этой хуйни


Для хуйни, пройдя по которой он передаст сигнал в котором содержится код доступа считывателю, а не для проца.
117269dSEXLZANPSed1288c29e2d697d422c7699366e3940.jpg133 Кб, 1024x1103
561 450411
>>50387

>Нет, смирись и найди себе другие развлечения


Okay.

>займись наукой.


Сложна.
562 450420
Какой же пиздецовый объем знаний в современной науке. Даже учебников и монографий дохрена, а уж публикаций и статей просто десятки тысяч. Как обращаться с таким количеством информации? Десяти лет не хватит, чтобы прочитать и разобрать все материалы даже достаточно узкой области, и ведь по 10 новых статей в день публикуют.

Неужели в современном мире невозможно быть полностью осведомленным о всех новостях и исследованиях в своей теме?
563 450421
>>50420
о всём более менее важным тебе журнашлюхи расскажут из каких нибудь околонаучный журналов. а там если тема заинтересует, то можешь и сам активнее первоисточники перерыть.

к тому же хоть информации и много, но её очень много однообразной. по той же астрофизики выходит тысячи работ в месяц, вполне серьёзных, но лишь единицы в год смогут как то серьёзно на что-либо повлиять.

нет нужды же их все изучать наизусть. ну найдёшь какую-нибудь звёздёчку интересную и захочешь о ней побольше узнать, но перед этим загуглишь, может уже кто написал о ней. Если данные уже есть, то можешь их уже сам изучить. Если нету, то сам работу пиши. Так же примерно и в остальных науках.
Тред утонул или удален.
Это копия, сохраненная 10 января 2019 года.

Скачать тред: только с превью, с превью и прикрепленными файлами.
Второй вариант может долго скачиваться. Файлы будут только в живых или недавно утонувших тредах. Подробнее

Если вам полезен архив М.Двача, пожертвуйте на оплату сервера.
« /sci/В начало тредаВеб-версияНастройки
/a//b//mu//s//vg/Все доски